Transcript for:
Devine Intervention: High-Yield OB-GYN for Third Year Shelf Exam

Okay, welcome. My name is Devine. I am a fourth year medical student and in today's episode of our Devine Intervention podcast, this will actually be a vodcast, we'll be reviewing some very high yield material for the obstetrics and gynecology third year shelf. So let's begin. I still have no financial disclosures. These are all just things I'll let you read on your own. It's just some advice for third year and some test taking strategies. So let's get started. So 22-year-old G1P1 female visits her obstetrician one week after she delivered a boy that weighed six pounds and four ounces. The delivery was unremarkable with Abgar scores of eight and nine at one and five minutes respectively, so pretty good. The patient plans to have her next baby when she graduates from graduate school two years from now. In addition to routine screening for postpartum depression, what is the next best step in the management of this patient? So, what do you think? Do we want to discuss the benefits of breastfeeding as an excellent long-term form of birth control? Do we want to do swabs for Neisseria and C. trachomatis? Do we want to give a combined OCP? Do we want to give Cetralin for postpartum depression? Or do we want to obtain consent for a DMPA injection? So what do you think? Good. So the answer is E. Okay, so this... A patient is postpartum, right? We clearly want her to breastfeed. And in general, if you want a lady to breastfeed, you don't want to give her any kind of oral contraceptive option that contains estrogen, so you do not inhibit breastfeeding. So the best answer here is E. So in the postpartum period, again, breastfeeding is pretty good, right? For like six months, but it's not completely reliable, okay? So on your exam, you definitely want to resist anything. that contains estrogen, resist the temptation to pick that answer choice. So for this lady, you want to give her progestin-only contraceptives. And in general, you want to remember the mechanism of action. It has many mechanisms, but the big ones that tend to be tested are the thickening of cervical mucus, progestin-only contraceptives protect against endometrial cancer. And if a person has a malignancy, a gynecologic malignancy, that is progestin receptor positive like a lobular carcinoma in situ or invasive lobular carcinoma you probably want to avoid a progestin continual contraceptive okay and remember that these injectable progestin analogs right it takes a pretty long time to return back to fertility okay in some cases they actually cause weight gain and they can actually cause a decrease in bone mineral density but it's usually reversible when you stop it the bone mineral density comes back and since we're on the topic of like breastfeeding and all that stuff i just decided to throw in that bonus there of tridymastitis with dicloxacillin. Now breastfeeding, right? Remember when you breastfeed, you have a very high prolactin state and prolactin shuts down the HPG axis, right? So if you're shutting down the HPG axis, you have less production of like stuff that can drive breast cancers, right? So you have a decreased risk of breast cancer. Hyperprolactinemia again shuts down the HPG axis. So you're not ovulating as much and When you're ovulating less, you're destroying that ovarian epithelial layer less and reforming it. Because think about it, when you ovulate, you explode out the egg, right? You repair the ovarian epithelium. You explode, you repair. You explode, you repair. So if you have less ovulation, you have less reparative cycles of that ovarian epithelium. So there is probably less risk of accumulating mutations and you don't get ovarian cancer. Okay? And actually breastfeeding actually helps with losing some of the weight you gain with pregnancy. Okay? But some big things you want to remember, especially for exams, if a lady has HIV, especially in a developed country like this, okay, or active TB or like active herpes lesions on the breast, that woman should probably not breastfeed, okay? And galactosemia, right, it's one of those biochemical disorders that can make their way to an OB-GYN shelf. Galactose 1-phosphate urinal transferase deficiency, you definitely should not breastfeed. It's a high-yield contraindication to breastfeeding. And then, because those kids don't want anything that contains galactose, which breast milk does. Okay. Now, mastitis. If a lady has mastitis, should she breastfeed? Good. She should. Okay. Because that will also help with clearing the infection. Okay. So, given the following clinical scenarios, what's the most likely diagnosis? So, a 22-year-old female is tearful three days after delivery. She has been breastfeeding and taking good care of the baby. What do you think? Okay, good. So these are postpartum blues. Just reassure the patient, try to shore up as much social support as possible, and she'll get better, hopefully soon. Next, the 22-year-old female is brought to the ED by her husband three days after delivery. Her found a small radio taped to her head. That's not a good sign. She claims to be receiving detailed instructions from outer space on how sacrificing her baby would end world hunger. So what is this? Right, so this is postpartum psychosis, okay? This is probably an indication for involuntary hospitalization. And you want to place this lady on an antipsychotic. Because if you don't involuntarily hospitalize this patient, there's a high risk of infanticide. Now, a 22-year-old female comes to her three-week postpartum visit. She looks disheveled and admits to occasionally having thoughts of hurting the baby, which she feels remorseful about. She has been breastfeeding the baby but no longer enjoys activities she loved before she got pregnant. She's accompanied by a husband who appears to be supportive, right? So this is postpartum depression. And in general, you want to start with an SSRI for treatment, okay? Usually, postpartum blues shows up at less than two weeks after delivery versus postpartum depression that is usually like two weeks or more, but it can show up in the immediate period after delivery. So don't hang your hat on that two-week rule, but it tends to work for most exam questions. So those are your answers. Now, next question of four four days after the cesarean delivery of a nine pound female a 33 year old g2 p2002 complains of abdominal pain vital signs include a blood pressure of 120 over 80 heart rate of 99 beats per minute a respiratory rate of 19 beats per minute and she's pretty febrile okay now physical exam is notable for diffuse lower abdominal tenderness and foul smelling vaginal discharge right so like foul smelling lochia A decision was made to pursue cesarean delivery secondary to an arrest of the active phase of labor. What is the most important risk factor for this patient's presentation? So what do you think? I'll let you think about it for a bit. Good, exactly. So the answer is E, okay? So this is a cesarean delivery. So what does this patient have? So fever after a cesarean section, what are you thinking about? endometritis. Very good, endometritis. Okay, so this patient has endometritis, and the biggest risk factor for endometritis is a cesarean delivery, right? So many of these other options, like multiple cervical exams, maternal infection prior to delivery, those are all risk factors for endometritis, but the biggest, the most important risk factor is a history of a cesarean delivery, right? So I'll encourage you as you go through your third year, or as you prepare for the shelf, you definitely want to sort of make listings of... high yield risk factors. Your OB-GYN people, the people at ACOG and the NBME, they love your risk factors, okay? So you definitely want to focus on that as you study. So this lady has endometritis. Again, the most important risk factor is the cesarean delivery, okay? And the common treatment regimen for endometritis on exams is a mixture of clindamycin and gentamicin, okay? The easy way to remember that is ECG, like getting an ECG, okay? So for endometritis, you give clinda and gent. Okay, any questions? Good. Okay, so please do not confuse endometritis with chorioamnionitis, okay? Chorioamnionitis is actually an infection of amniotic fluid, and the big risk factor you probably want to keep track of on your exam is a prolonged rupture of membranes. A prolonged rupture of membranes is probably one of the more important risk factors for chorio, right? Because the longer that those membranes are ruptured, right, the more chances you have for bugs to be introduced into the amniotic fluid. Okay. And I talk about the pathophysiology here. And basically you want to consider choreo. If you get an exam question where a mom has fever, okay, the baby has tachycardia, right? So normal infant heart rate is from 110 to 160. So if it's more than 160, think about fetal tachycardia. Okay. And then they tell you that, oh, the amniotic fluid is foul smelling. If you see that, think about choreo. Okay. And in general for choreo, your antibiotic regimen is like ampicillin and gentamicin. Okay, AMP and GENT, because the AMP helps you cover listeria pretty well. And you also want to sort of start proceeding towards delivery. You don't necessarily need to do a C-section, but you want to like give like IV oxytocin to sort of push the lady along with delivery. Okay, and an easy way to remember that antibiotic regimen is like CAG, like CAG repeats, like you have for Huntington's disease, right? So for chorionitis, you give AMP and GENT. Okay, and One quick word on listeria, right? So if a lady, if they tell you that a baby was stillborn, okay, and you find abscesses in multiple parts of the body, you should probably be thinking about grandparenting. pneumatosis infantiseptica, that's congenital listeriosis, okay? Listeria causes pretty nasty infections in neonates, right? You can actually kill the baby in utero. And then one weird thing I'm just going to go ahead and say here, it's just one of those bizarre scenarios that are very well written into nondescript questions, but you want to think about is if a lady has ruptured her membranes for more than 18 hours, okay, you probably want to go ahead and give her GBS prophylaxis, right? So like strep. a galacti, a group B strep. Now, how do you diagnose and date pregnancy, right? So obviously, I want to check the beta HCG, okay? You can check it in the urine or you can check it in the serum. The one in the serum tends to show up about a week before the one in the urine becomes positive, okay? And I will say that on your exam, you really should, like, I'll say, I mean, there is a chance this may not happen, but I will be very uncomfortable if you're finishing your MBME shelf. And you do not pick an answer that involves checking a beta HCG, okay? Something has probably gone awfully wrong on that test. So just be on the lookout, right? Because checking the beta HCG will be the right answer or the next step in management for many questions you see, right? So like a woman presenting with amenorrhea or like Sern's concerning for an ectopic pregnancy, we'll talk about that in a bit, okay? So a HCG, right, that's like the most reliable test for helping you indicate that, okay, maybe there's a fetus somewhere in this woman's body. And then if you want to calculate a lady's due date, if you know the first day of her last menstrual period, you add seven days, okay? You subtract three months, and then you add a year. That's called Nagel's rule. So let's do an example with that, right? So let's assume the first day of a lady's last menstrual period was August 8th, 2017, right? So you add seven days. That's August 15th, 2017. And then you bring back three months, right? So like August 15th to July 15th. July 15th to June 15th. And then June 15th. to May 15th, right? And then you add a year. So that'll be May 15th, 2018. So that's how you calculate the due date. Now, some other quick pregnancy basics, right? So you've, I mean, at this point in your rotation, you've probably heard of the TPAL system, okay? So if a lady is G4P3104, right? So the T stands for the number of term pregnancies, right? So she's been pregnant four times, right? So gravity means how many times have you been pregnant? So she's had three term pregnancies, okay? One preterm pregnancy, okay? Remember preterm is defined as anything from 36 weeks and six days or less, okay? And then she's had no abortions, that's what the A stands for. And then if you add up the last three numbers, that gives you the total number of living children, okay? Now let's play a game of who wants to be a pregnant millionaire. Okay, these are pretty easy, but some of them are a little difficult. But again, these are just one of those things you probably want to know for your exam. So, if a lady has a blue or purple cervix and vagina and she's pregnant, what is that? Okay, that's Chadwick's sign. Good. Now, what if she has a blotchy pigmentation of the face? Okay, that's the mask of pregnancy, right? Cloasma. Good. Now, what happens to the arterial blood pressure in the first 20 weeks? It should go down. Good. Okay. Because remember, pregnancy is associated with very high levels of progesterone. Okay. So those high levels of progesterone, progesterone is a smooth muscle relaxant. So it can cause like a decrease in systemic vascular resistance. So that can decrease blood pressure. Okay. After 20 weeks, your blood pressure tends to creep up a little. But again, it doesn't get higher than it was before you got pregnant. Now what happens to plasma volume? It goes up. Good. It goes up. Because remember, I sort of think of delivery as like a pretty bloody experience. So the body sort of prepares for all that bleeding that may happen by sort of sprucing up a mom's plasma volume. So plasma volume actually goes up by like 50%. Now what happens to the systemic vascular resistance? It goes down, again, from the progestin effect. Now oncotic pressures, what happens to oncotic pressures? What happens to oncotic pressures? Oncotic pressures actually go down, okay? Because your levels of albumin actually go down in the blood during pregnancy, okay? That's potentially why ladies, pregnant women have a lot of edema. Now what happens to cardiac output? It goes up, okay? Because your total blood volume is going up. So it's like you're giving mom's heart more preload. And if you go supine, what happens to cardiac output in pregnancy? It goes down, okay? Because remember, mom is being supine, so the uterus is potentially compressing the IVC, okay? That's why when a lady has, like, when a fetus has, like, a non-reassuring fetal heart tone, one of your next steps in management may be to tell the mom to lie on the left lateral, the cubitus position to sort of restore, to sort of boost the cardiac output. Now, what happens to your red cell mass? Okay, it goes up. Okay, it goes up by like 30%. But notice, I said your plasma volume goes up by like 50%. So if your red cell mass, right, a mass measure, right, so if you're thinking of concentration, your hematocrit is like a concentration measure, if you may. If your red cell mass is going up by 30%, but your plasma volume is going up by just 50%, what can you say about a lady's hematocrit, right? It should go down, okay? So that's physiologic anemia of pregnancy. And what happens to... coagulation factors, okay, they increase, right? So pregnancy is a hypercoagulable state. So it's a good thing and a bad thing, right? It's a bad thing because pregnant women are at increased risk of thrombotic problems, okay? But, and the way I think of it is the body saying, okay, this lady is going to bleed out a lot when she, I mean, not all ladies bleed out a lot when they deliver, but let's just assume, bleed out a lot when she delivers, right? So you may think, okay, how about we try to be more hypercoagulable? so that we can sort of clog off blood vessels, right, so that she doesn't bleed out and have like very bad complications, okay? So your levels of coagulation factors actually go up. In fact, I'll say in general, every protein in the bloodstream goes up during pregnancy, with a notable exception of albumin, for the most part. Okay, so what happens to gastric motility? I'll give you a hint. So high levels of progestin cause smooth muscle relaxation. So will your GI tract be chugging along nicely? Probably not, right? So gastric motility actually goes down, okay? So gastric emptying time actually goes up. Same thing for the colon. That's why moms tend to have a lot of constipation, okay? And they also have a lot of GERD when they're pregnant, okay? Now, acid-base imbalance. What kind of acid-base problem, at least mild, may you detect in the serum of a pregnant female? Alkalosis, right? An alkalosis. And why is that helpful? Okay, it's helpful, right, because if mom is alkalotic relative to the fetus, right, that creates a gradient for the fetus to dump carbon dioxide into mom's circulation across the blood placental barrier, okay, and tidal volume usually increases in the setting of pregnancy, okay. Now, the size of the ureters, let's assume the ureters contain smooth muscle, so what happens with progestin doing its job, right, the ureters dilate. Okay. So that's potentially why you have more urinary stasis and an increased risk of UTIs in a pregnant female. Okay. Now what happens to your BUN and creatinine? They both go down. Good. Right. So an elevated BUN or creatinine in a pregnant female is not good. Okay. That's a concerning sign. Okay. Because the GFR and creatinine clearance actually go up in pregnancy. Right. So think about it. If your blood volume is increasing, right, if you're thinking of your stalling forces, Okay. At the level of the glomerulus, right, your glomerular capillaries, your hydrostatic pressures are going up, right? So you're filtering more stuff out. So your BUN and creatinine should go down. Your GFR and creatinine clearance should go up because hydrostatic pressures are going up in the glomerulus. Now, what happens to the levels of glucose and protein in the urine? They increase, okay? They mildly increase, okay? Because the transport maximum for glucose at the level of the proximal convoluted tubule actually goes down in pregnancy. Okay, so some glucosuria and some very, very mild proteinuria is completely fine. Okay, now what happens to the size of the pituitary? So think of all those hormones mom needs during pregnancy. What happens? Good, so the pituitary should get bigger. Okay, it gets bigger. And if it gets bigger, right, I mean, it probably like doubles in size or more. Okay, if a lady bleeds out a lot during the process of delivery, okay, she can hypoperfuse that pituitary, or can basically have an ischemic stroke. of the pituitary gland, okay? That's pituitary, that's a She syndrome, right? So the classic exam presentation is like failure to lactate after delivery, okay? Now what happens to the size of the thyroid? It increases, right? Because again, you need thyroid, a lot of thyroid hormone for proper fetal development. So the size of the thyroid gland actually goes up, okay? Your thyroid-binding globulin, right? So estrogen actually increases the synthesis of thyroxine-binding globulin, So your TBG actually goes up. And because your TBG goes up, your total thyroid hormone levels, in fact, let me change this to total. So your total thyroid hormone levels actually go up. OK. But your free thyroid hormone levels are normal for the most part. OK. That's very right for like an arrow like question on an exam. OK. And what happens to your levels of beta HCG? They go up. So they increase like in the first. 10 weeks, they almost like double every two days. And then they sort of taper off and begin to decrease somewhat after 10 weeks. So those are your answers. Okay. So let me just modify this to total thyroid hormone levels. So total thyroid hormone levels. Good. Okay. So let this thing load. So let's keep going. Good. Now a 32 year old female at 16, at 36 weeks gestation comes to the ED because she has not felt her baby move for the past 36 hours. Physical exam reveals a distance of 37 centimeters from the pubic bone to the top of the uterus. Okay. No fetal cardiac tones are detected on Doppler ultrasound. That's not good. The rest of the exam is within normal limits. The patient's blood pressure is 105 over 78. Hot ready 78 bits per minute. Respiratory rate is 16 beats per minute and she's mildly febrile. In addition to patient-centered counseling and a demonstration of physician empathy, what is the next best step in the management of this patient? Give you some time to think about that. Good, exactly, right? So the answer here is D. Okay, so this lady has gone through an intrauterine fetal demise. okay so in this case uh i mean some of these answers let's just visit some of them real quick right so you want to tell mom to come back in seven days no right exactly right because if a dead fetus stays in the in the uterus for a pretty long time right uh that can uh cause a dic and sepsis and mom can actually die as well so you don't want to do that do you want to do a cesarean section no very good you don't okay the risk of the surgery is not worth it in fact i'll go ahead and see this right now If you have an intrauterine fetal demise, okay, resist every temptation on the exam to perform a c-section, okay? The risk of the surgery, not worth it, okay? So you want to do something that, I mean, it's not like you're going to tell mom, like, we got to deliver, like, right now. Like, no, it's only if it's, like, an emergent problem. But you sort of want to push mom towards delivering in some way, shape, or form, okay? So you want to induce labor with IV oxytocin. So again, deceased fetus, console the mom, respect their decisions, right? But you want to proceed towards inducing labor, right, for a vaginal. delivery. Okay, now, 25-year-old preemie gravida, 10 weeks gestation, presents for her first prenatal visit. Maternal exam and vital signs are within normal limits. Maternal hemoglobin is 11 grams per deciliter, and white count is 6,000. Most recent pap smear conducted five months earlier was unremarkable. Herr rubella titers are within normal limits, okay? She is heavy surface antibody positive, heavy core antibody negative, and heavy surface antigen negative, okay? chlamydia, syphilis and HIV screening, all negative, right? So things are all checking off so far. So what is the next best step in the management of this patient? I'll let you think about it. Do you want to continue prenatal care? Okay, maybe that's right, who knows? Right, so don't give vitamin A, right? It's a teratogen. Don't do that, okay? Do you want to give the intranasal influenza vaccine to a pregnant woman? What's bad about the intranasal influenza vaccine? It's live attenuated. Very good. So don't do that either. The NMR vaccine, very good. Same warning, right? So those are both live attenuated vaccines, right? So the answer actually here is urinalysis, okay? So again, don't be surprised if you see nondescript questions like this on the exam, okay? Basically, this question is asking about something that should be done very early in pregnancy, okay? You wanna screen for asymptomatic bacteria, right? So you do that with the urinalysis. So the thing is, because of the progestin effect causing like ureteral dilation and stasis, pregnant women tend to have an increased risk of UTIs. So you want to screen for asymptomatic bacteria urea because it can progress to pylo. Pylonephritis is obviously not good in pregnancy. In general, for every other kind of female that has asymptomatic bacteria urea, within reason, you generally do not treat. If you see bacteria in the urine but no symptoms, you don't treat. Okay. It's only if it's like complicated, but that's more for your medicine shelf. So we'll go there. But in general, a pregnant woman, asymptomatic bacteria, you want to go ahead and treat. Okay. And you can treat it with like nitrofurantoin or amoxicillin. You can give phosphomycin. Okay. And just as an aside, if a lady has pylo in pregnancy, you want to give IV ceftriaxone. Okay. And a weird thing is once a lady has pylo nephritis, it actually increases the risk of like preterm delivery. Okay. But the thing is, you actually want to maintain that lady on suppressive therapy, like with nitrofurantoin, for example, for the rest of pregnancy. Okay. And after you treat the lady for pylo, okay, or for the UTI, you want to go ahead and perform a test of cure. Okay. You want to perform a test of cure. Okay. And again, remember, pregnant females, they need folate, so you don't have a neurotube defect. And don't give excessive vitamin A, right? That's like not a good thing. Okay, now, what is the most appropriate time frame for the following prenatal tests slash interventions, right? So, RHD immune globulin in an RH negative female. Very good, at 28 weeks, okay. Now, vaginal swab for ruby strep, 35 to 37 weeks, okay. When you want to screen for gestational diabetes, okay, at 28-ish weeks, okay. Now, when can you do CVS, chronic valence sampling? Between 10 to 13 weeks. Very good, okay. And a screening ultrasound for neurotube defects. around 18 to 20 weeks right and then an amniocentesis quad screen good after 15 weeks awesome okay so uh i put the approximate timelines here it's just one of those things you probably want to know okay and uh because they'll usually present it as a question about a lady that's worried about her kid having some kind of um problem like chromosomal problem right and then they'll give you a series of tests and they'll give you like the weeks of pregnancy in the question and you have to figure out like okay 10 weeks, I probably cannot do an amniocentesis, right? So, but one thing, I don't know if it's necessarily made its way to the MBMEs yet, but it probably will within a couple of months or a few years is cell-free DNA. You can actually go ahead and get that at about 10 weeks, okay? And basically, if you do any of these invasive procedures that could potentially mix mom and baby's blood, you also want to go ahead and give mom RHD immune globulin, even if it's not 28 weeks, okay? Now, if AFP is elevated in mom's serum, right, you want to think about a neural tube defect, right? Usually, those may also have an elevated acetylcholinesterase in the amniotic fluid, okay? But you should also probably think of an abdominal wall defect like omphalocele or gastroschisis, okay? Although, probably the most common cause of an elevated AFP is incorrect deeding, okay? Now, if you find an increased level of inhibit A and beta-HCG, right, then AFP and estriol are down, okay? You also want to think about like... Down syndrome, right? Trisomy 21. Remember that can arise from like a maternal non-disjunction or like a robotronium translocation. I talk about that in a different podcast, okay? For trisomy 18 Edwards syndrome, basically your AFPS trail and beta HCG will all be low, okay? And they may describe it as a kid having like rocker bottom feet and like overlapping digits and toes on an exam, okay? And if a quad screen is positive on your test, your next step in money gain generally is to get some kind of ultrasound, okay? And just don't forget your basic science-y step one detail, where if your lecithin to sphingomyelin ratio is two or greater, okay? That's indicative of fetal lung maturity. And then for gestational diabetes, right? I said you can screen between 24 to 28 weeks, okay? In general, you perform like a one-hour oral glucose tolerance test. If it's positive, you then proceed to a three-hour oral glucose tolerance test, okay? It's like the confirmatory test, if you may, okay? I probably won't memorize the code of values. They'll almost, I mean, I can't say for sure, but I would imagine they'll give you those values on your exam. But, excuse me, you want to know how to interpret those, okay? And in general, for gestational diabetes, you can try like diet, exercise. But insulin is probably like first line, at least on exams, okay? But ACOG is actually beginning to feel more comfortable with like metformin and glyburide. So that'll probably, again, make its way to MBME territory soon. And one thing I'll just say here, so I don't forget it, just came to mind right now, is that if a lady has a history of, like, second trimester, like, painless pregnancy loss, right? So, like, from cervical insufficiency, you can potentially place a cerclage, okay, for that female, okay? But the thing is, actually, having a cerclage is a contraindication to exercise. OK, so in general, we don't discourage pregnant women from having from exercising. But once a surclash has been placed, that's a very high yield contraindication to exercise. So just keep that at the back of your mind. OK, now, why do pregnant women get gestational diabetes? Right. It's probably from having human placental lactogen. It's a diabetic hormone. OK, and it serves good purpose. Right. Its primary job is to create like a hyperglycemia in mom so that she can shunt more energy to the fetus. OK. And gestational diabetes, it has certain problems it can cause, right? So like big baby polyhydramnios, right? So if a baby is constantly seeing glucose from mom's blood, the baby will also have high... his own hyperglycemia and people that are hyperglycemic right they make more urine right so the kid can be uh can be uh you the mom may have a polyhydramnios right so like it's an afi greater than like 25 and then uh when the kid is withdrawn from mom right after the baby is delivered right so the baby is no longer seeing those high levels of glucose but the pancreatic cells are like wait They're still secreting as much insulin as they normally do, right? Because they're used to hyperglycemia. So that can actually trigger a hypoglycemia in the baby, and that can actually trigger a seizure, okay? So hypoglycemic seizures are a potential complication of an infant of a diabetic mom, okay? You could also have seizures from hypocalcemia, but I don't have the time to talk about the mechanism right now. Now, one thing I want you to understand is, there is sort of a difference between the complications that... a baby gets if mom develops gestational diabetes versus the complications that a baby gets if mom has diabetes before she even gets pregnant, okay? So if a lady has like chronic diabetes, right, there are certain things you actually want to keep in mind, right? So those kids, they can have cardiac problems, okay? The classically tested one is hypertrophic cardiomyopathy, okay? They can have fusion of the legs, that's like syrenomelia. They may also have like their sacrum doesn't develop like right, so they could have like a caudal regression syndrome. And again, hypocalcemic seizures. And one other thing I'll just say is if a baby, like the estimated fetal weight is more than 4,500 grams, you probably want to recommend a C-section, right? A planned C-section. Because the risk of cephalopelvic disproportion is a little too high at that point. Okay. Now next question, right? So this is just straight memorization, unfortunately. So given the following maternal BMIs, what's the recommended weight gain during pregnancy? Okay, so let's take it in order, right? So if a lady is underweight, how many pounds can she gain? 20 to 40, 20 to 40. Okay, now if she's normal weight, so BMI 23, 25 to 35. Okay, if she's overweight, 15 to 25. Very good. And if she is obese? 11 to 20. Very good. Okay. So unfortunately, you just need to memorize that. Okay. So given the following scenarios, what's the most likely diagnosis, right? So a 39-year-old female presents with mood swings and amenorrhea for the past year. During the interview, she describes having severe episodes of intense sweating and palpitations once or twice daily. Herr medication list includes levothyroxine and physical exam reveals skin hyperpigmentation. This is kind of a bizarre question, but let me give you some hints here. If a lady is taking levothyroxine in the US, what does she potentially have? Maybe Hashimoto's? Okay, so that's an autoimmune disease, okay. Skin hyperpigmentation. Addison's disease, very good, right? So, she may have Addison's, okay. Remember... You have high levels of ACTH and it comes from POMC and POMC also creates MSH, right? So you get skin hyperpigmentation. So if a lady has these postmenopausal-ish symptoms and she's 39 and she has all these autoimmune problems, good, right? So this is premature ovarian failure. Very good. This is premature ovarian failure. Awesome. Now, a 19-year-old female presents with intense anal pruritus. Physical exam reveals erythema of the vulvovaginal area. She recently completed a course of Cipro for low-bar pneumonia. She has a history of type 1 diabetes and a pH of a virginal secretions of 4.3, right? So it's less than 4.5. And KOH prep reveals spores and structures resembling hyphae. What is this? Candida. Very good. Candida. Okay. So Candida, remember the pH... Remember your big associations, right? So like cottage cheese discharge from a lady's vagina on an exam, okay? And the pH is, again, less. You want to remember that 4.5 cutoff. Candida has the less than 4.5 pH. But trich and Gardnerella vaginalis, right, in bacterial vaginosis, the pH is usually greater than 4.5, okay? And there are certain high-yield risk factors for Candida vaginitis, right? So if a lady is immunosuppressed for any reason or she has like... diabetes, okay, or recent antibiotic use, okay, those are very high-yield risk factors for candida vulvovaginitis, okay, and in general, you can give an antifungal cream or like orofluconazole, okay. Now, bacterial vaginosis, right, caused by Garnierilla vaginalis, the big clue there is a fishy smell from the vagina, okay, on KOH prep, okay, and you'll see clue cells, and for that, you give metron, okay, you'll tell the lady taking metronod to take, um... take alcohol, right, because you don't want to have the disulfiram effect from the inhibition of acetaldehyde dehydrogenase. And you don't need to treat the partners for candida or gadnerella, okay, but for trich, okay, you want to treat the lady and her partner, okay, to prevent recurrence. And you also treat that with the metro, right. So remember your get gap on the metro mnemonic from step one for the bugs that can be treated with metronidazole, right. So like the G stands for giardia, okay, the E is for entamoeba. Remember that can cause a bloody diarrhea. The T stands for trich vaginalis, okay? Trich vaginalis, like your frothy, green discharge, foul smelling, and then you may see like pontate hemorrhage on the cervix, right? So your strawberry cervix, if you may, okay? The other G, so get gap on the metro, so the G stands for gadnerella, right? So like bacterial vaginosis, the A is for anaerobes. Remember, metronidazole classically treats anaerobes below the diaphragm, okay? And then the P is for protozoal infections, okay? Now, with menopause, because the ovaries basically stop working, okay, your estrogen goes down, but your FSH-LH levels go up because there's no negative feedback, okay? And some treatments for menopause, right? So if the lady says, doc, it really hurts when I'm having sex, okay, you want to give like a lubricant, okay? You can give vaginal estrogen to sort of build up that epithelium down there, okay? The vaginal estrogen, though, if a lady has a history of like an estrogen responsive malignancy, you probably want to go ahead and avoid that. OK, now for osteoporosis, because remember, estrogen, right, increases the synthesis of osteoporotegin and osteoporotegin sort of binds up rank ligand. So you have less activation of osteoclasts. If a lady has osteoporosis because she's hypoestrogenic from menopause. OK. You probably want to supplement calcium and vitamin D. You may want to give a bisphosphonate. A bisphosphonate is actually the first-line medication for osteoporosis. You can also give raloxifene. Remember, raloxifene is one of those serums. It's an estrogen receptor blocker in the breast and in the uterus, so you don't have an increased risk of endometrial cancer. But it's actually an estrogen receptor agonist in bone, so it's actually protective against osteoporosis. Now, here's one weird thing that they love to use to mess with your head on like the OB-GYN shelf. or the medicine shelf, okay? Or potentially a peds or psych shelf as well. So if you're obese, you naturally bear weight, okay? Being obese is actually a protective factor against osteoporosis, okay? So if a lady has anorexia, right, she's not bearing weight, she actually has a pretty high risk of osteoporosis because she's also hypoestrogenic. But obesity, the more weight you bear, the more we are in turn, you're... joints, okay, you actually have an increased risk of osteoarthritis. So obesity does, in fact, increase risk of osteoarthritis, but it actually decreases the risk of osteoporosis, okay? Make sure you understand those, okay? And I say some things like if you have like super severe menopausal symptoms, you can give like a short course, like a six-month course of hormonal replacement therapy, right? You want to tag on progestin with that, right? So that you don't have, you don't run into problems. Although, if a lady has a history of like strokes, or like a migraine with aura, right? Or she has like a history of like an estrogen responsive malignancy or like a liver mass, like a hepatic adenoma. You probably want to go ahead and avoid those altogether. Okay, next question. A 34-year-old female, sorry, at 10 weeks gestation is brought to the ER by her husband, who is concerned about his wife having severe vomiting for the past week. Physical exam reveals skin tenting and a fall of greater than 20 millimeters of mercury in the patient's... systolic blood pressures with changes from a supine to a standing position. Herr weight today is 131 pounds. Okay. Herr weight is recorded as 140 pounds in the chart three months ago, right? So before pregnancy. Further testing of this patient would most likely reveal what? Good, right? So this is a hypokalemic, right? Hypokalemic metabolic alkalosis. So what does this patient have? Hyper-MS is gravidarum. Excellent. Very good. Hyper-MS is gravidarum, right? So she has the, she's, excuse me, she's vomiting a lot, right? So she's losing a lot of acid. So she becomes hypochloramic because she's, again, vomiting acid, she becomes metabolically alkalotic, okay? And because she's volume down, she increases the activity of her renin-angiotensin-aldosterone system, right? So that Enec transporter at the level of the principal cell works a lot because aldosterone is high. Okay, so you extrude more potassium in the urine, so you become hypokalemic, okay? And also, if you increase the activity of that proton ATPase pump at the alpha-intercalated cell of the nephron, okay, you also can develop a metabolic alkalosis from that because you're dumping more protons in the urine, okay? And she has lost more than 5% of her pre-pregnancy weight, right? So like 5% of 140, right? So 10% is 14, 5% is 7, good. So she's lost more than 5% of her pre-pregnancy weight, okay? So that qualifies her for hyper-MSC's gravidarum. So, one quick thing I'll just pull off from here is hyperhidromyces gravidarum may actually be a presentation of gestational trophoblastic disease, okay? And in general, you want to give these people, like, because they usually volume down, you give them IV fluids, you can give them, like, ondansetron, right, or metoclopramide, right, to... to prevent emesis, right, to reduce the vomiting, and then you want to give them the electrolytes that they're missing. An NBMA favorite appears to be like vitamin B1, thiamine, so just sort of keep track with that, okay? But if a lady has like morning sickness or pregnancy, just tell her to like eat small frequent meals, and then take like vitamin B6 and doxylamine, okay? Like Unisum, basically. Okay, next question. A 26-year-old female presents to obstetrician for a yearly checkup. She has a past history of hypertension that has been well controlled with Captopril. and lifestyle modifications. Blood pressure is 120 over 80. Herart rate is 65 beats per minute. Respiratory rate is 15. Other vital signs and the physical exam are within normal limits. BMI is 23. The patient had an Mirena IUD implanted 18 months ago and wants it removed at this office visit since she got married three months ago and plans to have kids. The last pap smear was two years ago. In addition to routine preventive guidance and recommendations on appropriate vitamin supplementation, what is the next best step in management? I'll give some time to think about this. Good. Very good. So the answer here is E. Okay, you want to switch captopril to alpha-methyl-Dopa. OK, you want to switch captopril to alpha methyl dopa. OK, so let's look at the other answers here real quick. So a pap smear. Right. So this lady had a pap smear two years ago. Right. If you're between 21 to 29 years. Right. The recommendation is a pap smear every three years. So she had one two years ago. So one is not indicated now. OK. Captopril. is an ACE inhibitor. Are ACE inhibitors good for pregnant women? No. Good, right? It's a teratogen. Do you want to switch to erbicartan and ERB? Good. You don't want to do that either, right? Because again, ACE inhibitors and ARBs, right, they can cause a renal damage in the fetus, okay? GBS prophylaxis is not indicated, okay? This lady is not at 35 to 37 weeks and she's not even pregnant to start with, okay? But the right answer is actually to switch from an ACE inhibitor to alpha-methyl dopa. Okay, so remember that alpha-methyl-dopa is one of the antihypertensives that are safe in pregnancy. Okay, you probably remember your step one mnemonic that hypertensive moms love nifedipine for the safe antihypertensives in pregnancy, right? So like hydralazine, that's the H, M for methyl-dopa, alpha-methyl-dopa, love, the L is for labetalol, remember it's an alpha-beta blocker, and then the N is for nifedipine, okay? That's... dihydropyridine calcium channel blocker, okay? So the answer is E, those are safe in pregnancy, okay? And I've talked about that. And actually on the next slide, I will go into some of the gradations of hypertension, right? They love you to know this for your exam. So chronic hypertension is you're pregnant before you're, sorry, you're hypertensive before you're pregnant. It's pretty straightforward, okay? Now, gestational hypertension is more than 20 weeks of pregnancy, your blood pressure is more than 140 over 90, okay? No proteinuria at that point. But if you take that hypertension and alprotinuria, then you jump into the land of preeclampsia. No one really knows the pathophysiology of preeclampsia, but there are certain buzz phrases you probably want to recognize for tests. Endothelial dysfunction is one of them. Formation of abnormal blood vessels, that's another thing you want to think about. And as always, you want to know your risk factors. You want to know your risk factors. There are many risk factors for preeclampsia. I have them listed here. But the most important risk factor, okay? for preeclampsia is a prior history of preeclampsia okay that's the biggest risk factor very high yield to know that now you can then jump from preeclampsia to preeclampsia with severe features where they tell you that the person has like signs of some kind of end organ dysfunction right so like lfts are going up b and creatinine are going up that's usually like not a good sign okay alternatively if a lady's blood pressure is more than 160 systolic over 110 diastolic she qualifies for preeclampsia with severe features, okay? And in general, at that point, you probably want to begin to consider a mag for seizure prophylaxis, okay? Although, I'll just take a quick detour here and tell you that you want to do like regular physical, like a regular physical exam, right? While the lead is in L and D, because you want to check the deep tendon reflexes, right? So a decreased DTR, deep tendon reflex, is like a telltale sign of potential mag toxicity. You probably want to turn off the infusion. Okay. If you don't watch it at that point and it continues, okay, you go to respiratory depression and then the lady could have a cardiovascular collapse and die. Okay. Get like a nasty arrhythmia and like die. So you don't want to do that. So in general, magnesium, the telltale sign of magnesium toxicity is a decrease, the deep tendon reflex. Okay. And in general, if a lady has marked toxicity, the rescue agent is actually calcium gluconate. Okay. And other treatments, right, for the hypertension, you can give like labetalol or nifedipine. But once a lady has preeclampsia and she has seizures, boom, that's eclampsia. At that point, you give MAG, not a benzo. If MAG does not work on an exam, then you could consider giving a short-acting benzodiazepine. But MAG works better than benzos for seizures. And after that, you really want to proceed to delivery. That's the only... definite treatment for eclampsia, okay, delivery of the fetus, okay, and then HELP syndrome is like hemolysis, elevated LFTs, and low platelets, okay, so they'll usually give you like a CBC to point you in that direction on exams. Okay, so given the following birth defects, what's the most likely teratogen, okay? So, hypoplasia of the right ventricle and downward displacement of the tricuspid valve. In fact, I'll give you the buzzword, atrialization of the right ventricle. And let's assume this mom has been treated for, the mom of this baby has been treated for a bipolar disorder. So this is lithium toxicity, Epstein anomaly. Okay, good. And I'll just add this here that actually one of the first line meds for treating mania in pregnancy is actually a Haldol. Lithium is still good, but haloperidol, at least as per our friends at UpToDate. Now, tooth discoloration. Good, right? So tetracycline, right? Like doxy. Now cartilage damage, chloroquine alone hurts your bones, right? So that's like Cipro or Levo or whatever. Stippling of the epiphysis and don't use this anticoagulant in a pregnant female with a DVT. Good. Okay, good, right? So that's warfarin. And I just want to see this now that it's come to mind. I'll just throw this in here. I hope I'm not wrong, but I think, no, it just came to mind. I think if a lady has a history of preeclampsia, you could potentially give her like aspirin prophylaxis with her next pregnancy. I'll have to check on that, but I'm pretty sure that's actually correct. If it's wrong, I'll probably just update that in a different podcast. Now, clear cell adenocarcinoma of the vagina, that's DES, right? So diethylstilbestrol, okay? Remember, it's also associated. T-shaped uterus. Now, renal problems in the fetus. We've talked about this already. ACE inhibitors or ARBs. Good. Gray baby syndrome. Chloramphenicol. Awesome, right? Because kids, like neonates, for example, they tend to have very low levels of UDP glucuronacetal transferase, right? So they cannot metabolize the drug, right? So it can cause problems for them. Now, crinicterus. TMPSMX. Very good, right? Back trim. Now, smooth philtrum, you should never get this wrong on an exam, it's pretty classic. Smooth philtrum, that's all I need to say. Fetal alcohol syndrome, good. Now, IUGR, hypoplastic nails, microcephaly and a cleft lip, that's phenytoin, okay? That's fetal hydantoin syndrome, very good, okay? And what's the most dangerous, I guess, the anti-epileptic that's associated with the highest risk of neurotube defects? Valproic acid, good. Remember, it has liver toxicity. Okay, so those are your answers. Now, a 21-year-old female presents to a PCP with complaints of severe facial acne. Herr last menstrual period was one week ago. The patient is in a stable relationship with her boyfriend of three months, and they use condoms inconsistently. Physical exam is notable for open and closed comedones clustered around the patient's lower face. Prior attempts with tetracycline, benzoyl peroxide, and combined OCPs have failed to yield any positive results. The patient smokes two cigarettes per day and requests a prescription for isotretinoin. The most likely contraindication to this treatment regimen is what? Good, right? So isotretinoin is a teratogen, right? So the answer here is C, right? So the patient's history of unprotected sexual intercourse, right? So in general, if a lady is placed on isotretinoin, she needs to be on two forms of birth control. Excellent. Okay, and I'll just say some quick things here. I've talked about most of the teratogens, but remember your anti-epileptics with neurotube defects, okay? If a lady is 35 and she smokes, she should not be placed on a combined OCP. If a lady has a history of, like, migraine with auras or some kind of hepatic adenoma or history of thromboembolic disease, again, any OCP that contains estrogen, not good for her, okay? Now, if they're described by a kid, aww. Umm... If you describe a kid that's jittery, a newborn that's jittery, inconsolable, you want to think of some kind of withdrawal like opioid withdrawal. Usually you give methadone for that, like a long-acting opioid, and then you taper that off. And then cocaine used in pregnancy, it can cause abrupt placenta. That usually presents as painful third trimester bleeding. And then in general, you probably want to use more acetaminophen in pregnancy, especially after 32 weeks. If you begin to use like indomethacin. Remember, that's a Cox inhibitor, right? So you decrease the synthesis of prostaglandins. That can actually trigger premature closure of the ductus arteriosus, which is obviously a bad thing, right? Next question. A 29-year-old G2P becomes unresponsive 15 minutes after delivering a 3,900-gram newborn. Vital signs include a blood pressure of 40 over palpable. This is really bad. Herart rate of 40. That's not good. Respiratory rate of 6 and an O2 sat of 78%. The attendant notices blood oozing from a venipuncture site. What is the most likely diagnosis? This is just something I want to recognize on a test. Good. Right. So this is an amniotic fluid embolism. Awesome. Okay. Now let's do some breast questions. Okay. So let's do some breast triggers. So 44 year old male being treated for New York Herart Association class three heart failure presents with gynecomastia. What is the drug that improves survival that may cause gynecomastia? Spironolactone, very good. Okay, it's an aldosterone receptor antagonist, but it also has androgen receptor blocking activity, so it could cause gynecomastia. Okay, now 32-year-old postpartum female, so four weeks after birth, presents with fever, breast tenderness, and erythema. Asthitis, awesome. Okay, how do you treat that? like loxacilin right so your anti-staphylococcal penicillins okay uh 23 year old female presents with multiple mobile breast masses these masses become more painful and wax and wean in size with a menstrual cycle so multiple it's not single it's multiple young female good those are fibrocystic changes okay now 23 year old female solitary mobile well-defined painless breast mass you That's a fibroadenoma. Awesome. Bloody nipple discharge. Good, right? This shows up on every shelf, right? That's an intraductal papilloma. Okay. Now, a 49-year-old female presents with a large breast mass, and a corneal biopsy reveals a mass containing structures with leaf-like projections on histology. That's a phylodes tumor. Good. Now, what's the BRCA mutation that's more commonly associated with male breast cancers? BRCA2. Awesome. Now, what's the most common location of breast cancers? Upper outer quadrant. Okay. And what's the receptor positivity most commonly found in LCIS? Good. Many of them are estrogen and progesterone receptor positive. Awesome. Now, what's the most common form of invasive breast cancer? Invasive ductal carcinoma. Okay. Now, what if you have a 55-year-old female that presents with a breast mass and a physical exam reveals port-orange changes? Inflammatory breast cancer. Awesome. Now, 55-year-old female presents with an eczema. Scaly appearing lesion around the nipple. Patient's disease of the breast. Awesome. Now, is there any difference in survival between lumpectomy plus radiation or mastectomy for early stage breast cancer? No. Good. Now, what's the next best step in management of a female with DCIS who has a positive sentinel lymph node biopsy? So the sentinel lymph node biopsy is positive. You want to do an axillary lymph node dissection. Good. Now, how do you treat ER, PR, positive breast cancers? Tamoxifen. Tamoxifen. Or you can also use anastrozole. That's an aromatase inhibitor. So you have less conversion of androgens to estrogens. Now, how do you treat her two new positive breast malignancies? Treated with Herrceptin. Good, right? Trastuzumab. Awesome, Trastuzumab. Now, for the 5-year-old female with a history of breast cancer treatment, presents with JVD and an EF of 35%. She's been treated for breast cancer, and now she has heart problems. So, heart problems, Herrceptin maybe? Good, that's Trastuzumab. Remember, Trastuzumab can cause a dilated cardiomyopathy that tends to be reversible. Okay. Contrast that with another set of drugs that could be used to treat breast cancer. Your doxorubicin. Those are cardiotoxic, right? So cardiotoxorubicin. Those cause dilated cardiomyopathy, but it's usually irreversible. Okay. In general, you can try to prophylax against that with an ion chelator, dextrozoxane. Okay. Now, a 55-year-old female with a 10-year history of lymphedema from an axillary lymph node dissection presents with weight loss. and what appears to be a purplish necrotic ultradian mask on her left arm. Yeah, this one is kind of hard. It's a lymphangiosarcoma. Okay, it's a lymphangiosarcoma. Good. So those are your answers. Now, breast cancer, it's floridly high yield for your exam. Okay, you probably shouldn't take your exam if you don't know much about breast cancer. And the thing is, it's very scenario based. Okay. In fact, some of these scenarios I'm going to pose, the answers are kind of like, you'll hear like different I would say answers from different sources, but these are my best judgments as to what I've seen with ACOG and I've seen online. But I'll advise you to look some of this up some more. But I'm relatively certain of the answers I placed here. If anything sort of bugs you, just sort of send me an email in the comment section and I'll try to do some more research. But I did pretty good research. when I was writing this up. And I tried to be as vague as possible because that's how they tend to write these questions. So let's talk about some scenarios. So 45-year-old female presents with complaints of a palpable left breast mass she noticed when she was taking a shower one week ago. Okay. What is the next best step in the management of this patient? Do you want to do a corneal biopsy first? No, right? You want to go ahead and do a mammogram first, okay? A mammogram. You want to do, if you're palpated breast mass, you usually want to jump to some kind of imaging first, okay? And because she's more than 30 years old, you want to do a mammogram. Okay. Now, a 45-year-old female presents with complaints of a palpable left breast mass she noticed when she was taking a shower one week ago. A diagnostic mammogram is negative. What is the next best step in management? We'll show the patient. No, very good. No, they love to mess people's heads with this on exams. OK, if you palpate a breast mass on exam, OK, and the imaging is negative, that does not rule out breast cancer. You need to proceed to a corneal biopsy. OK, if you can palpate it, do imaging first. But if the imaging is negative, that does not rule out breast cancer. You need to proceed to a corneal biopsy. Now, 25 year old female presents with breast pain. Herr last menstrual period started three days ago. physical exam is notable for multiple mobile breast masses what is the next best step in the management of this patient tricky tricky we get an ultrasound what do you what do we think this lady has fibrocystic changes right very good right so they're pretty common in females like this and this is a question that's like sort of pretty classic for fibrocystic change okay so what do you do for this lady you Good, you reassure her and full up in a few weeks. Okay, awesome. Now, a 25-year-old female presents to a yearly gyne appointment with complaints of a palpable breast mass she noticed while taking a shower. Physical exam reveals an immobile, that's concerning, painless mass located close to the nipple. She has no family history of breast cancer. What is the next best step in the management of this patient? This is probably a fibroadenoma, right? So, do you want to reassure her and let her come back in a few weeks? No, good. Why do we not want to reassure this lady? Even if she's young and it's almost probably not breast cancer. Good, right? It has some concerning features, right? So it's immobile, okay? It's immobile. It's immobile, okay? So with this, what do you want to do? You want to get some kind of imaging, obviously. So what do you want to get? Mammogram? No, good. No mammogram, okay? So this lady, she's less than 30 years old, okay? Younger females tend to have like more dense breasts, okay? So you want to go ahead and get an ultrasound in this scenario. Good. Next question. 25-year-old female presents for a yearly gyne appointment with complaints of a mass she felt while taking a shower. Physical exam reveals a palpable, immobile breast mass in the upper outer quadrant of the breast. Uh-oh. Ultrasound reveals what appears to be a solid mass. What is the next best step in the management of this patient? A solid mass. Good, you want to get a corneal biopsy, okay? Solid masses on ultrasound, corneal biopsy, okay? Now, next step in management, what if you saw a cystic mass? Want to do a corneal biopsy? No, good. You want to get an FNA, okay? You want to aspirate the fluid. You may send it for cytology. But usually, once you withdraw that fluid, you can full up in a few weeks with like a full-up ultrasound, okay? Now, what if you withdraw fluid and it was bloody? It was bloody. Okay, that's worrisome. Okay, so you can send the fluid for cytology, but you clearly want to go ahead and do a biopsy of that, a corneal biopsy. Okay, now another question. For the same patient from the previous slide, what is the next step in management if a prior ultrasound with FNA revealed serous fluid and a new ultrasound at this visit reveals a recurrence of the cyst? Right, so remember the first scenario I said. Cystic fluid, you did an FNA set follow up in a few weeks. This is the follow up appointment, you see the recurrence of the cyst. What do you want to do now? Drink it and tell her to come back again. No, very good. No, you want to go ahead and do a corneal biopsy here. Okay, recurrence of a cyst, that's kind of worrisome. You want to go ahead and get a corneal biopsy. Okay, so I will highly recommend going through these scenarios. I think you'll pay off in spades on your exam. Okay, so those are your answers and I've sort of explained my reasoning beyond that already and also added a few extra details here. Okay, now some other high-yield things you want to know about breast cancer, right? So there are two kinds of screening guidelines, right? There's the American Cancer Society one where you perform like a screening memo starting at the age of 40 every year. And then there's the USPSTF one that's probably more conservative, right? Where they say start from 50, go up until 74, do it every two years, okay? But I would just say, and I've actually seen both tested, but I'll say the one that appears to be more reliable in OB-GYN land, okay, on exams, tends to be the American Cancer Society one. So I'd probably go with that if I were you. Now, instead of saying corneal biopsy, they may also actually make an answer choice excisional biopsy. If it's like a small, like super well-circumscribed mass, since you're already doing a biopsy, just go ahead and cut out the entire thing. Okay. And women that are more high risk, right? So like women that have like BRCA mutations, in addition to the mammograms, they generally get like MRIs of the breast. Okay. And remember, you can do a total prophylactic, a total, and I won't repeat this sentence again. So I'll just say it once. Now, if you ever see me write THBSO, it just means total abdominal hysterectomy and bilateral salpingo-oophorectomy. Okay. Because you want to prophylax against like breast, ovarian and endometrial cancer. Good. Next question. A 33-year-old G2P11 female presents to L&D at 34 weeks for evaluation of consistent uterine contractions. Herr last pregnancy required a classical C-section, secondary to severe obesity. As the OB-GYN resident walks into the room, the patient begins to... So notice, classical C-section. I'll let you think about that for a bit. So as the OB-GYN resident walks into the room, the patient begins to complain of severe abdominal pain. A pelvic exam is notable for... copious amounts of blood emanating from the vaginal canal. Fetal monitoring reveals a heart rate of 33. That's bad. The patient has a history of cocaine use with a current pregnancy. What is the most likely diagnosis? Yeah, this question is tricky, tricky. What do you think? The first thing I would encourage you to think about here is history of a classical C-section, okay, and categorize this patient's presentation as painful or painless. It's painful. So could it be A or B? No. Good. So what do you think this is? Good. This is uterine rupture. Okay. A classical C-section is one big risk factor for a uterine rupture. Okay. And if you see like the trend of thought on the question, right? So like very sudden onset, sudden like just really bad fetal heart rate tracing, and you see like severe abdominal pain. In a lady with a history, again, of a classical C-section, you really want to think about a uterine rupture. And really, what you want to do here is like a crushed cesarean section. You want to get the baby out as rapidly as possible. I tried to mess with your head a little by putting history of cocaine use to get you thinking of abrupt placenta, but that's not even an answer choice here. And other things you probably want to think about here. I didn't put the classic buzzwords you're used to seeing on exams, right? So they may say like, oh, that the baby was at plus two station and now the baby is minus one station, right? So loss of fetal station. That's a classic exam presentation of uterine rupture. Another thing you may want to think about is they tell you that they are palpating like weird masses in the abdomen. OK, that's the basically palpating fetal parts. OK, so that's a uterine rupture. So. If a lady presents with third trimester bleeding, the first question you want to ask yourself on exams, okay, are if it's painful or painless. If it's painless, you want to think about either placenta previa, where you have like a low implantation of the placenta, or You want to think about a vasoprevia, where basically the baby's blood vessels are overlying the cervical os. If it's painful, you want to think about a uterine rupture, or you want to think about abruptio placentae. Abruptio placentae on exams, it's painful, but they'll usually give you a mom with a history of risky behaviors, like cocaine use. They may also, but if they give you the kind of presentation like I gave you for those buzzwords, for the question, you want to think more about a uterine rupture. So that's how you sort of parse those two apart, okay? And in general, for visaprevia, you want to go ahead and deliver like super soon, okay? Because it's an obstetric emergency. Now, let's go to the land of postpartum hemorrhage, okay? So according to our friends at ACOG, okay? I actually enjoyed my OB-GYN rotation. According to our friends at ACOG, okay, if you lose more than 500 mils or cc's of blood... after a vaginal delivery or a thousand cc's of blood after a c-section, that's postpartum hemorrhage. Now, there are many causes of postpartum hemorrhage. The most common cause is uterine acne. Lacerations could also cause postpartum hemorrhage. They're probably the second most common cause. Retin-placental tissue could also cause postpartum hemorrhage. and then DIC, like the amniotic fluid embolus I talked about, and then urine inversion, okay? Let's think of that as a med student problem. We'll talk about that in a later slide, okay? And one easy way to remember this, this was actually thought to me by a... I believe he's now a fellow. Yeah, he's an MFN fellow. Her actually taught me this mnemonic. Okay, so like tone, tear, tissue, thrombin, and topsy-turvy to help you remember the five common causes of postpartum hemorrhage. Okay, so what can cause uterine acne, right? So a few things can cause uterine acne, right? So think about it. If you work out for like 10 hours, you probably like to die at the gym. Okay, so the uterus is overworked from like putting in superhuman force in rapid labor, or you are like in labor for like forever, right? If you're expending too much energy, that can cause, make the uterus sort of like give up. If the uterus is infected, right, that can also cause uterine acne. If the uterus is too relaxed, right, so if you give too much like tributylin or rhododrine, like too many tocolytics, that can over-relax the uterus, make it lose its tone, okay? Or if the uterus is too distended, right, just... too much going on right so like multiple gestations like if a lady's carrying triplets obviously the uterus is like obviously overworked uh polyhydramnios right so like an afi greater than 25 or like big baby right like an infant of a diabetic mother okay this can all cause uterine acne now in general for uterine acne the first step in management is you perform a uterine massage under ultrasound guidance and you give iv oxytocin to contract down the uterus you can also do a procedure called you put place a billing sutures It's basically like suspenders on the uterus. And then you can actually give tonic agents. Oxytocin is probably the most common one used. But you could also use methogen. The thing is, methogen is a serotonin receptor agonist. Okay? So if a lady has a history of vasospastic disease, like Prince-Meadows angina, I think they've changed the name to like Virenth angina. Or if a lady has... Um... Uh... Raynaud's phenomenon, right, so let's say for example from like Crest scleroderma, you probably don't want to give hermethargin, okay? That's why you don't give tryptans to those patient populations as well because tryptans are serotonin receptor agonists, okay? Now you could also give carboprost, okay? But it causes bronchoconstriction, so don't give it to a person with a history of asthma, okay? It's called like hemobate, right? Like hemobate, right? So a bait bleeding, if you may. And then dinoprostone is a PG2 analog. In addition to being useful for opening up the doctor's arteriosus, for keeping the doctor's arteriosus open. it's also useful for uterine acne, okay? But you want to avoid it in women with a history of like, if they're like profoundly hypotensive, you probably don't want to do that because it's a powerful vasodilator. Now, lacerations, right? So if you have like, if the baby just pops out really quick, you can imagine that many things will tear down there, okay? If you also have a difficult vaginal delivery, right? So like shoulder dystocia, that can cause a laceration. If you also have like an operative vaginal delivery, right? That can also increase the risk of a... a lack, which can cause postpartum hemorrhage. And you treat that with surgery. Now, for retained placental contents, right, so if you have, like, a succincturic lobe of the placenta, right, so it's like an accessory, like, tiny lobe of the placenta, you may bring out the normal placenta and say, like, oh, it's all done, but there may be some stuff still in there. Or if a lady has, like, invasive placenta, we'll talk about that in a later slide, that can, again, cause, actually, like, life-threatening hemorrhage. And in general, you just do an ultrasound and sort of curatage the retained placental contents carefully. Now, DIC, I've sort of talked about that. I've given a question on that, so I'll let you go through that on your own. And then a uterine inversion, okay? There are many things that cause it, right? So, like, the myometrium is weak. You pull too hard on the uterus. The big risk factor here, actually, is a history of a prior uterine inversion, okay? And the classic exam presentation is they'll say, like, oh, like, they've been trying to pull out the placenta. And then you see like a bluish bulge, right, like a bulging blue mass or a bulging mass coming from the vagina. Okay. That's pretty classic for uterine inversion. Okay. In general, the way you want to fix it is you literally like box the uterus. I mean, you don't box it, right, but you replace the uterus. And then you give IV oxytocin to contract down the uterus. Okay. So this is my schematic of uterine inversion. Okay. So this is normal uterus. You pull too hard. Let's say you're a med student that's super like eager. Don't worry, residents could also cause this problem, or even attendants if you may. So, you pull too hard, you invert it. So how do you fix it? You replace the uterus, and then you give it your teratonic-like oxytocin. Now, if all else fails, right, you basically begin to ligate arteries, right? Like the uterine arteries, the internal iliacs. And if all else fails, you take out the uterus, okay? So you get a hysterectomy. Now, if you bleed too much after pregnancy, remember the pituitary, like, increases in size, like a lot. during pregnancy, so the mom can hypoperfuse the pituitary and she can infarct the pituitary, okay? That can be She syndrome. So think of She syndrome as an ischemic stroke of the pituitary gland. I know they tend to try to confuse this with pituitary apoplexy. This is like a rudimentary explanation, but think of pituitary apoplexy as a hemorrhagic stroke of the uterus. I'll talk about I mean of the pituitary. I'll talk about that in a different podcast, okay? But think of She syndrome as like an ischemic stroke, and then think of pituitary apoplexia as like a subarachnoid hemorrhage, if you may, of the pituitary gland. I'll talk about that in a different podcast. Now, this is just a summary of everything I've talked about, so I'll let you go over that on your own. Now, menorrhagia, right? So menorrhagia, there's actually hard definitions, believe it or not, of this stuff, but... Patient perception, let's get real here. Patient perception is the more important thing, okay? So if a lady is just burning through a turn of pads, she probably has abnormal uterine bleeding, okay? And the differential diagnosis, there's a palm coin mnemonic that is classically used in the world of OB-GYN. Although if you unscramble the letters, it reads policeman, so whatever floats your boat, I guess. But I probably stick with palm coin if I'm you, because it helps you separate these out as structural and non-structural causes. of abnormal uterine bleeding. Okay. And we'll go into these as we proceed with this presentation. But before I go into those, let me just... Excuse me. Let me talk about some quick alphabet soup here. So metrorrhagia means you're bleeding between periods. So sort of think of a metrobus coming and going pretty quickly. And then polymenorrhea, so usually a normal time between menstrual periods is like 21 to 35 days. If you're less than 21 days, that's polymenorrhea, too many menses. If you're more than 35 days, too few menses. That's oligomenorrhea. And then if you're having like regular menses, but your flow is super light, right? So like probably like less than 80 cc's, if you may. That's hypomanoria. I mean, I guess probably 80 cc's is not correct. But if your flow is too light, that's hypomanoria. And then if you combine those terms, menometroragia, right? It's like your bleeding is irregular, right? So like you're having irregularly time periods, but you're also bleeding pretty heavily. So let's talk about some causes of abnormal uterine bleeding, right? So the first one I want to talk about is a leiomyoma. Lay people refer to this as fibroids, okay? So basically, this presents as asymmetric nodularity of the uterus, okay? It's usually like painless. Well, I mean, it could be painful in some cases, but it's really painless, and the uterus is usually asymmetric, and it's usually firm, okay? It's usually firm. If you notice, I'm making these buzz phrases known, right? So asymmetric. painless, firm. Contrast that with symmetric, painful, soft. Okay, symmetric, painful, soft, symmetric, painful, soft. That's what goes more with adenomyosis. I'll talk about adenomyosis in a bit, okay? And in general, you make your diagnosis, you do a good physical, you can usually palpate this on exam if it's big enough, okay? And you can also detect them on a transvaginal ultrasound. And basically, the way you treat that is you use like OCPs, you try to regulate the the HPG axis, okay? And you can also provide NSAIDs if it's painful, okay? Now, here's probably where your meat is on exams. If a lady has leiomyomas, right, and she wants to get pregnant in the future, okay? Because actually, leiomyomas are one cause of uterine factor infertility, okay? You want to perform a myomectomy. You don't want to do a hysterectomy, okay? But if the lady is like a postmenopausal, want to get rid of this uterus, you can go ahead and perform a hysterectomy. And usually before surgery, you can give continuous GnRH like Lupron, like Luprolide. That will shrink the tumor because these leiomyomas are usually estrogen responsive. Now, other etiologies, right? So you can, I've talked about Lupron, I've talked about NSAIDs. You can supplement iron if you have iron deficiency anemia. You can give tranexamic acid. Think of it as a clot-busting inhibitor. okay so that the lady bleeds less okay and i'll talk about some other specific causes in later slides now if a lady is bleeding like a lot right you want to do your usual things right like type and cross place large bar ivs to replenish fluids give fluids transfuse as needed iv estrogen is probably your first line especially if a lady has a uh bleeding that is and she's hemodynamically unstable okay iv estrogen can fix that pretty quickly but if all else fails uh you probably shouldn't be handling that on your own Go ahead and call your friendly OB-GYN resident or attendant, okay? They'll do all these fancy-schmancy things they can do and rescue you there. Now, next question. 27-year-old G0P0 presents for a routine GYN annual. She has no history of abnormal pap smears. Over the last decade, she has had irregular periods. She has been married for the past three years, and despite having unprotected intercourse with her husband every other day, has failed to have any children. Physical exam reveals male pattern hair growth around her chin with no evidence of clitoral megaly. Herr BMI is 35 and her blood pressure is 150 over 102. She requests a referral to a dermatologist for a long-standing hyperpigmented lesion she noticed under her breast and axilla six months ago. What is the most likely diagnosis? I'll just give you all the buzzwords here. Irregular periods, right? So dysfunctional uterine bleeding, male pattern hair growth, no kids, infertile, big BMI, hypertensive. acanthosis nigricans, right? So this is PCOS. Good. So this is PCOS. So PCOS, how do you make your diagnosis of PCOS? PCOS is a clinical diagnosis, okay? You actually do not necessarily need imaging if you have enough signs. on a good physical exam, right? So your three criteria, and you probably should, actually, you should go ahead and memorize this criteria, right? So polycystic ovaries on ultrasound, that's one, okay? If you have some kind of evidence of hyperandrogenism, right? So like if a lady's hirsute, right, that's one thing to think about. If she has some objective sign of anovulation, okay, you just need two out of three, and you can make the diagnosis, right? So the thing is, there are other things you can find with PCOS, right? So like signs of insulin resistance. or LH to FSH ratio that's elevated, but those are not part of the diagnostic criteria, but they can support your diagnosis. So, in general, right, if a lady, uh, presents like this, you also want to try to rule out other causes of hyperandrogenism, right? So like, I'll let you read this on your own. It's not very relevant to our discussion. And there's a pathophysiology of PCOS. People really do not know what causes PCOS. Well, there are some thoughts, and I tried to organize the literature as best as I can into this summary diagram. I'll let you go over that on your own. I'll say probably the big thing you want to pick up from here for your exam, I like the treatment options, right? So like clomiphene, I'll talk about why that is. I'll talk about combined OCPs in a later slide. Metformin weight loss, right? So the things in red are pretty good for the treatment of PCOS. I'll talk about that shortly. So, what could happen with PCOS, right? So, if you have long-standing PCOS, you could have, like, metabolic syndrome, antecedents, right? So, like, stroke, MI, blah, blah, blah. You could become infertile. In fact, this is actually the most common cause, very high yield, most common cause of ovulatory factor infertility, okay? It can actually cause endometrial hyperplasia or cancer. And actually, let me stop there and see something here real quick. In general, if a postmenopausal female presents with bleeding, From the vagina, right? You want to think about endometrial cancer. So you want to do like an EMB, an endometrial biopsy with sampling. But here's one thing you want to be wary of. On your exam, they could try to trick you, right? So this is something I would imagine they may want to put on a test, right? So if they give you a question about a lady that's like 40, right? Or like 39, okay? They may not even say she has abnormal bleeding, okay? But they may tell you that, oh, she has like a five-year history of PCOS. And they may say, oh, what is the next best step in management? They may say like they've done some things for her in the question. Your next step in management, you probably want to think about an endometrial biopsy as well, okay? Because the hype... Hyperestrogenism that accompanies PCOS actually very highly increases risk of endometrial cancer. So even if they are not postmenopausal, you want to go ahead and do that screening. Now, how do we treat PCOS? So different goals. If you want to reproduce, you can consider clomiphene. So clomiphene is a serum. It's an estrogen receptor antagonist, although think of it more as a partial agonist. So if you remember from step one, when you learn about beta blockers like... acebutalol and pindolol. Acebutalol and pindolol are partial beta agonists, but because they're not giving you the full catecholamine effect, right, they're regarded as beta blockers, okay? So that's sort of how clomiphene works, right? So by blocking estrogen receptors in the central nervous system, right, you can increase GnRH pulsatility because you're basically shutting down estrogen's negative feedback, right? So that actually promotes ovulation and that can encourage fertility. Now, you can also try to regulate the HPG axis, right? If the lady doesn't want to have, like, inconsistent bleeding, you can do that with OCPs. They have, like, multiple mechanisms. They regulate the HPG axis. But another thing they do is estrogen increases the synthesis of many proteins, right? So, like, just like I said with thyroxine-binding globulin before, it actually increases the synthesis of sex hormone-binding globulin. And SHBG is actually a pretty good testosterone binder. And if your testosterone levels go down, you can decrease the hirsute or hyperandrogenic symptoms you get. PCOS. Now, you could also give spironolactone, right? So, spironolactone, it does help you with many things, right? So, two things I highlighted here is it can help with the hyperandrogenism you get in PCOS by blocking androgen receptors, okay? And it actually helps with the hirsutism because it inhibits 5-alpha reductase in the skin, okay? Remember, 5-alpha reductase converts testosterone to DHT, okay? So, by inhibiting 5-alpha reductase, you could basically um... help with the hirsutisima symptoms. And then for the insulin resistance, right, you try to lose weight, give metformin. That's what I'm going to say here. And then for other agents, these are particularly low yield. So I'll just let you read that on your own for your own information, but I'm going to keep going. So next question, a 32-year-old female presents to L&D at 37 weeks gestation with consistent moderately painful uterine contractions. Herr pregnancy has been complicated by a history of diabetes. Fetal heart tracings are unremarkable. The patient was offered a C-section secondary to fetal macrosomia, but insisted on having a natural birth. Okay, that's usually not a good sign. I mean, natural birth is good, but read on. During the second stage of labor, the baby's anterior shoulder is stuck under the pubic symphysis. The baby is successfully delivered after repeated traction on the shoulder and arm, in addition to applying maneuvers involving hip flexion in the mom. So this baby's delivery increases his risk of injury to what? So what do you think? This is a very nice integrated question. So, Erbdusheen Palsy, right? So, what are the nerve roots connected with the pathophysiology of Erbdusheen Palsy? Good! C5-C6, okay? C5-C6. So, remember the waiter's tip deformity, right? So, the kid's hand is like pronated, the elbow is extended, and the... The wrist is flexed, okay? So that's Herrb Duchenne Palsy. So shoulder dystocia. It's actually an obstetric emergency. You want to handle this pretty quickly because it's bad, because you can have like fetal hypoxia. So in general, there are certain... things you can try right you can apply like suprapubic pressure um you can also do something called the mcrobert's maneuver so basically tell mom to flex her hip okay and tell her to push uh there is one other maneuver i'm forgetting where basically mom goes on all fours uh and then And basically, if all of that fails, you go ahead and do something called the Zavanelli maneuver. I have never heard of the Zavanelli, at least I've never seen the Zavanelli maneuver done. In general, it's something you generally want to try to avoid because it's just not very ideal. Talk to an OB-GYN, they will explain that to you. But basically, a Zavanelli maneuver, you basically like get the child back into the uterus and perform a C-section. That's usually not ideal. Okay, but usually these maneuvers usually work pretty well. Okay, and... And please don't be tempted with Klumke's palsy, right? C-8-T-1, that's more for a neuro shelf. And I've talked about cephalopelvic disproportion, so I'll keep going. So let's go to a fancy fetal heart rate retracement. Fetal heart rate retracement, this is something that usually causes people a lot of stress and anxiety. But it's actually pretty simple. Okay. Hopefully in the next like three minutes, you should feel good about this stuff. Okay. So let's talk about a normal heart rate tracing. Okay. So this is a normal heart rate tracing. Okay. The average. fetal heart rate is like 110 to 160 bits per minute, right? So if it's less than 110, that is fetal bradycardia. If it's more than 160, it's fetal tachycardia. That's the first fact you want to know, okay? Now, in general, you want to see some squiggles. You can see some nice squiggles here, right? You want to see some kind of variability. Okay. And then you want to know what defines an A cell, an acceleration. Okay. If you hear me say A cell, I mean acceleration. If you hear me say D cell, I mean a deceleration. Okay. So an A cell basically is, remember your rule of 15s. Okay. A rise in heart rate of 15 bits per minute for 15 seconds. Okay. That is an A cell. So your rule of 15s. Increase in, just like you know your twos for your Meckel's diverticulum, remember your rule of 15s for Robiguy, okay? So an increasing heart rate of 15 beats per minute for at least 15 seconds qualifies as an A cell, okay? And if you have two of those in a 20-minute period, that is a positive non-stress test, okay? If you have a positive non-stress test, you're basically reassured for the fetus's heart rate, okay? So you can continue with expectant management, if you may, okay? But... If a non-stress test is non-reactive, right, so you don't see two A cells in a 20-minute period, you want to proceed to something called a biophysical profile. It's a more complete test, if you may. And there are five components to the biophysical profile, okay? It's an NST, okay? You check the amniotic fluid volume. If it's less than 5, that's oligohydramnios. If it's greater than 25, that's polyhydramnios, okay? And then you want to check, like, freedom of breathing, movement, and tone, okay? And basically, you score two for each. The max score is 10. If you're 8 to 10, you can probably, again, continue expectant management. But if your numbers are low, 6 is sort of a gray area, but if it's like 4, you probably want to proceed towards delivery of some sort. So let's do some fetal heart rate tracings here. So all of them will be abnormal. But they're like the high-yield scenarios you want to be able to ID on an exam. So the first one is, given the following fetal heart rate tracing, what's your diagnosis? So let me point out the big thing here. So we have... a contraction, okay, and then we have this gradual decrement in the fetus's heart rate, and then a gradual return to baseline, okay, gradual decrement, gradual return to baseline, and they coincide with a contraction of the mom's uterus, okay, so what is this? This is what? An early deceleration, very good, okay, so this is an early decel, okay, it's it's fine okay it's not i mean it's not normal but uh it's actually pre it's it's okay it's benign okay so this is an early decel okay and it's caused by head compression okay i'll give you the best mnemonic in all of obi-gyn to remember this at the end okay so this is an early decel caused by head compression okay and again the thing that defines an early decel is contraction pairs up with the decrement in the fatal heart rate and the fatal heart rate decrement is gradual so with a gradual return to baseline. And I'll summarize all of this at the end as well. This is a reassuring fetal heart rate tracing. Now let's try this one. So let me point out the big things, okay? We can see all of mom's contractions, okay? But we can see that the fetal heart rate goes down abruptly and very quickly comes up back to baseline. So it goes down abruptly, comes back up pretty quickly to baseline. And if you notice, um... Yes, some of these decrements in heart rate conflict with the mom's contractions, but some of them don't fall perfectly along with that. What is this? So this is a variable D cell. Very good. This is a variable D cell, okay? And the thing that defines it is not whether the decrement in fetal heart rate, excuse me, traces along nicely with the uterine contraction. It just means that you have an abrupt drop in fetal heart rate. and a very quick return to baseline, okay? This is generally not reassuring, okay? And for this, you want to think about cord compression, umbilical cord compression, okay? You generally want to maybe proceed towards some kind of delivery with this as well because this is a non-reassuring fetal heart rate tracing, okay? Now, notice this one. This one, you have the contraction being, right? So if you remember your physics from college, don't worry. Don't get a panic attack with that. But basically, this waveform is out of phase with this waveform, right? So again, you have a gradual decrement, a gradual return to baseline. But these decrements in the fetal heart rate are... out of phase with mom's uterine contractions, this is what defines a late decel, okay? It's a late decel, so it's a gradual downslope of the fetal heart rate, okay? And again, the downward slope is out of phase with the maternal contraction. For this, you want to think about some kind of utero-placental insufficiency, okay? The thing is, a classic question on the exam is, they will say, what's your next best step in management? If you see this, the first thing you want to do is you want to increase perfusion of the placenta, so you want to turn mom. to a left lateral decubitus side, okay? And then after that, you want to sort of try to proceed towards some kind of delivery, okay? So how do you differentiate between all the different fetal heart retracements, right? So let's, so we say that, oh, if you have, so basically just the first question you should ask yourself is, if you think fetal heart retracements are normal, the first question you should ask yourself is, is there an abrupt... an abrupt drop in the fetal heart rate, okay, and a very quick return to baseline. If you see that, it's more, it's most likely a variable decel, okay. But if you see a gradual decrement in the fetal heart rate and a gradual return to baseline, you then ask yourself one question. Is the decrement in fetal heart rate in phase, does it match up nicely? Does the trough of the fetal heart rate decrement match up with the... crest of mom's uterine contraction if it matches okay that's most likely an early d cell if it does not match if it's out of phase then it's a late d cell okay so just think of that as a quick algorithm for resolving fetal heart rates on your exam okay so remember uh normal fetal heart rate is 110 to 160 a positive an a cell is 15 seconds 15 writes of 15 beats per minute okay and a positive non-stress test is two a cells in a 20 minute period 2 and 20, okay? And then, if you think of an abnormal fetal heart rate tracing, ask yourself, is there an abrupt drop in the fetal heart rate and an abrupt return to baseline? That's a variable D cell, okay? If you have a gradual decrement and a gradual return to baseline, ask yourself, is it in phase with mom's uterine contractions or out of phase? If it's in phase, that's an early D cell. If it's out of phase, it's a late D cell. An early D cell is fine. a late D cell or a variable D cell and non-reassuring fetal heart rate retracement. And then this one, this one is just pattern recognition. This looks like a sinusoidal wave. What is this? Good. This is fetal anemia. This is bad news, right? So usually for this, you may want to do like a... POPs, like a percutaneous umbilical blood sampling, to check the baby's hematocrit for anemia. The good thing about that procedure is you can also actually do a blood transfusion at the same time, okay? So this mnemonic is ridiculously awesome, okay? I highly recommend that you commit this to memory. It almost always pays off on exams, on OB-GYN exams particularly, okay? So it's called VLCHOP, okay? So the V stands for variable D-cells, okay? That's caused by cord compression. The E stands for early D cells. It's caused by head compression, okay? The A stands for A cells. A cells are good, caused by nothing. That's the O. And then the L stands for late D cells. It's caused by uteroplacental, that's the P, insufficiency, okay? It's a very useful mnemonic. It'll probably get you a few points on your exam. Now, next question. A 15-year-old, so how do you guys feel about fetal heart rate traces? Good? Awesome. Now, a 15-year-old primigravida presents to a PCP with complaints of severe morning sickness and vaginal bleeding. She has been vomiting for hours every day and has lost over 15 pounds since her last menstrual period eight weeks ago. She has been taking Benadryl to help with the nausea. Blood pressure is 140 over 98. Respiratory rate is 18. Herart rate is 103. A transvaginal ultrasound reveals a central heterogeneous mass in the uterus having a solid hyperechoic area. interspersed with a multitude of cystic areas. Beta HCG is positive. What is the next best step in the management of this patient? What do you think this is? Severe morning sickness. There is a weird OB-GYN pathology that can actually present as severe morning sickness, or like hyperemesis gravidarum. What do you think it is? So what's your answer? Bingo. Good. So suction cure attached. So what does this patient have? Molar pregnancy. Very good. So this patient has a molar pregnancy. And I tried to trick you here. So the thing is, usually the MBM is pretty nice, or your friends at ACOG are usually pretty nice. Let's not say usually. Let's say 50% of the time. They'll give you the buzzwords you're looking for. Snowstorm appearance on ultrasound. If you see that, obviously this question becomes easy. You know it's a molar pregnancy, end of story. But basically what I did is I took the words snowstorm appearance and just described them for you. Like the solid hyper-equaic area interspersed with blah, blah, blah. That's basically code word for snowstorm. Another classic MBME trick is... If they say laminate calcifications, that's another like a descriptive term for a samoma body, right? So for that, right, you probably want to think about like a papillary thyroid cancer or meningioma. Okay, remember the association with that with neurofibromatosis, okay? It usually shows up around the folk cerebri. And then if you're thinking more OB-GYN, that's laminate calcifications, samoma bodies. You want to think of the ovarian cancer like serocyst adenoma or serocyst adenocarcinoma. Okay, so very high yield to know those. So, this is a molar pregnancy, so suction curettage. Now, so this patient has a hydratidiform mole, okay, they arise from a trophoblastic content, okay, and there's actually two types of hydratidiform moles, okay, there's the partial moles and there's the complete moles. Don't let the word partial fool you, okay, you may think, oh, partial, it must be worse. No, it's not. Partial moles actually, I mean, they're bad still, but they're actually better than complete moles, okay, and how does a partial mole arise? A partial mole usually arises when one sperm, okay, no, sorry, two sperm, two sperms, okay, fertilize one egg, okay? So you form a triploid embryo. Those are the potential genotypes, okay? And in general, partial moles actually have some fetal tissue, okay? And they produce a ton of HCG, but not as much as the complete moles, okay? And they could theoretically progress to an invasive mole, okay, or progress to a choriocarcinoma, but In general, I will say that they have much lower risks of that kind of progression in comparison with a complete mole. Okay. So partial moles, triploid, a ton of HCG, but not as much as complete moles. Okay. And they have a very miniature risk of progressing to an invasive mole or choriocarcinoma. But contrast that with a complete mole. A complete mole, it's a lot worse. Okay. It's diploid. So usually what happens is two sperm fertilize an empty egg or one sperm fertilizes an empty egg and then it duplicates its genetic material. Okay, unlike a partial mole that has fetal tissue, a complete mole has no fetal tissue, okay, and a complete mole tends to produce a lot more beta-HCG than a partial mole, okay, and it has much higher risks than a partial mole of progressing to an invasive mole or and choriocarcinoma, okay, and in general on exams, I'll say this is probably a pretty classic finding, is that molar pregnancies usually present as size greater than dates, right, so let me see how late he is. 17 weeks pregnant, and then you measure from the pubic symphysis to the top of the fundus, and you're measuring like 21 centimeters, okay? That's pretty concerning for, and you see like this in this clinical context, the first thing that should probably come to your mind is molar pregnancy. So you want to get an ultrasound to see the snowstorm appearance, okay? And hyperamesthesis gravidarum may also be a presentation because remember, that's why in the first 10 weeks of pregnancy, right, like the first trimester, ladies have like very bad morning sickness, right? Because They are vomiting a lot. And why is that? Because they're having very high levels of HCG in that first trimester, okay? So because these tumors secrete a ton of HCG, right, they present with a ton of vomiting, okay? And if you're just in, the patient has a molar pregnancy, you perform a suction curettage, you want to place that patient on birth control for like six months, okay? You want to follow the HCG values to zero because you're trying to watch against like recurrence. Now... Now, two months after a suction cure attached, the patient begins to have vaginal bleeding and shortness of breath. Beta HCG is super high. What is the next best step in management? What do you want to get here? An ultrasound? I mean, you may get an ultrasound. You probably will get an ultrasound. What do you want to get first? Shortness of breath. A chest X-ray. Very good. A chest X-ray. Okay. So, this basically is choriocarcinoma. Okay. What's the most common metastatic region of a choriocarcinoma? The lungs. Very good. The lungs. It could actually go to the brain in some people. So you want to get a chest x-ray. So this is Corio. In general, you should have methotrexate. Remember, that's a dihydrofolate reductase inhibitor. If they give you a question and you say, oh, the lady has severe bone marrow suppression from methotrexate, you want to add on locovorin. It's a folinic acid analog. Now, Corio, you can actually present as hyperthyroidism because remember, beta-HCG and TSH, they actually share a common alpha subunit with also, I think, FSH and LH. So they have that common alpha subunit so Choro can actually present as hyperthyroidism but in this case TSH will be low and your thyroid hormone levels will be high. Yeah, okay, I'll have to think about that for a second. Now, if you detect chorea and you treat it, okay, you want to place the lady on birth control for a year, right? So for like a molar pregnancy, six months, for chorea, you want to place her on birth control for a year, okay? You want to train the beta HCG levels down to zero, right? So that, because if she gets pregnant, you may begin to ask yourself, hmm, is it? from the fetus or is it from recurrence of cancer, right? That's why they should like not be pregnant for a while. Now, given the following case scenarios, what is the next best step in management? Okay, so 29-year-old G2P11 female presents for her first prenatal visit at 10 weeks gestation. She's RHD negative, okay? Herr anti-D immune globulin titer is negative. What is the next best step in management? It's negative. What's the next step in management? What do you want to do at 28 weeks? Good. You want to give RHD immune globulin. Awesome. Okay. Now, assume a similar patient has that in the first vignette, but the patient is RHD negative as well, but they have a positive anti-deimmune globulin titer. What do you want to do next? And now, this is a tricky question. What is the very less invasive thing you can do with a blood test? Who do you want to test? The dad. Very good. You want to check... That's RH status, right? Because if that is RHD negative, okay, then there's a 0% chance, basically, of the baby being RH positive. So you don't need to worry about anything here, okay? So there are more things you need to do, right? So let's assume that is RHD positive, right? Or that's RH status is unknown, right? Then you want to do a couple of things, right? There are some very complicated algorithms for this, but let me give you the CliffsNotes version, okay? The CliffsNotes version is that... You want to do certain things like you want to check for fetal anemia by doing like a Doppler ultrasound of the middle cerebral artery of the fetus. Because think about it, right? If you're anemic, you become hypoxic, right? So your body says, hmm, if you're thinking of those cardiology equations you learned while studying for step one, the body will say, okay, let me try to increase cardiac output. Okay, so you can have like more blood flow, right? So if you have increased flow through the middle cerebral artery that you detect on Doppler ultrasound of the fetus, okay, you're probably worried about anemia. Okay. And there are certain things you want to do, like you want to like maybe do if you detect anemia, you may want to do pulps and give like percutaneous umbilical sampling and give like blood transfusion, blah, blah, blah. But that's probably a little too detailed for the shelf. But I'll say that if you sort of know the things I talked about, you're probably good to go with that. You can do some more reading on your own as well. Now, a 35-year-old female with a history of four C-sections presents with... painless vaginal bleeding at 32 weeks gestation. What is the next best step in management? So this is painless vaginal bleeding. So what do you think this is? It could be placenta previa. Very good. What else could it be? Especially if the kid has like a nasty fetal heart rate tracing. Good. It could be vasoprevia as well. Okay. But we really don't know. But I want to illustrate a general principle here. If a lady presents with third trimester bleeding, do you want to go ahead and poke the vagina for starters? No, good, right? You want to go ahead and do an ultrasound first, okay? You want to go ahead and do an ultrasound first, okay? Why do you want to do an ultrasound first? The reason is if the patient has placenta previa or vesa previa, for example, and you go and poke those fetal, like, so let's say vesa previa, right? You do a vaginal exam, you poke those blood vessels, okay? And then the lady bleeds out and the baby bleeds out and maybe dies, okay? That's not ideal, right? So you want to, like, do an ultrasound first, okay? To make your diagnosis first or make sure that... Everything around the cervical os is like, okay, before you then go for the vaginal exam, okay? And in general, right, so painless vaginal bleeding, they're one of two things you're thinking about, right? So you may want to think of placenta previa, especially given the mom's history of, like, many C-sections. It's like four, that's kind of a lot. You want to think about placenta previa, right, because think about it. If you have multiple C-sections. You may have like many of those C-sections, they may heal with like scar tissue. So the placenta is like, oh, scar here, scar here, scar here, scar here. Where do I implant? It just implants like lower uterine segment. That can be placenta previa. That's actually the biggest risk factor, very high yield to know that. The biggest risk factor for placenta previa is history of a C-section. Other things could cause it. So like if you have like multiple gestation, so think about it. If there are like four kids down there, right? It's like, oh, I got it. grab this region of the uterus for placenta, right? If a kid has grabbed one spot, the other kid may say, okay, I guess I'll manage this, right? And then they pick different spots, right? One of them could have a lower uterine placental implantation, and that can be a placenta previa, okay? And for placenta previa, basically what you want to do is, if it's pretty early, like if you detect it at like 30 weeks or like 31 weeks, whatever, you need to proceed to delivery now, okay? Because sometimes the placenta can actually change position, okay? So that's one thing that could happen. But some high-yield things you probably want to keep in mind is you want to tell mom to like stop having sex, okay? So that's what pelvic rest basically means. You want to administer steroids for fetal long maturity, okay? And if you're less than 32 weeks, you want to give mag. Okay, because MAG actually decreases the risk of cerebral palsy, right? So the buzzword you're looking for is neuroprotection, okay? And basically, if it has not resolved by like 36-ish weeks, you want to go ahead and do a planned C-section, okay? You don't want mom to like have a vaginal delivery and potentially have a life-threatening bleeding that may actually like kill her or the baby or both. Okay, so next question. Given the following clinical scenarios, what is the next best step in management or the correct diagnosis? Okay, so 37-year-old G2P11. with a prior history of C-sections, presents with severe bleeding after the second stage of labor, ultimately requiring a hysterectomy, intra-op evaluation of the uterus revealed an adherence of the placenta to the myometrium. So what is this? Good, placenta or critter. What if the placenta invaded the myometrium? That's placenta in critter. Good. Now what if it invaded the bladder? That's placenta per critter. Okay. Now, 23-year-old female G2P11 is rushed to the OR after fetal monitoring, revealed multiple persistent lead decels. Herr pregnancy has been uncomplicated. The OB-GYN resident noticed severe vaginal bleeding after her membranes were ruptured. So notice the sequence of events. Painless vaginal bleeding, really bad fetal heart rate tracing, the membranes ruptured, and then all this badness started happening. What do you want to think about first? Good. Visa previa. Visa previa. Okay. Basically, you want to do a crash cesarean section. Okay. That baby needs to be out like yesterday. Okay. You want to proceed to a C-section without delay. Because this is actually an OB-GYN emergency. It's actually pretty morbid if nothing is done quickly. Now, a 32-year-old G2P1, one of 34 weeks gestation, is the only survivor of a severe motor vehicle accident. She's brought to the ED by local ambulance. She complains of abdominal pain. Transabdominal ultrasound is unremarkable. Pelvic exam reveals blood emanating from the cervical os. This patient has a history of polychondral blood pressures and heavy cocaine use. Oh no, her last to be going visit was at 10 weeks gestation, right? So she didn't have regular prenatal care. So what do you think this is? Painful vaginal bleeding, mom used cocaine, polychondral blood pressures. Good. Abrupt your placenta. Awesome. Awesome. Okay, you generally probably want to do a c-section for that. Now, 25 year old G2P11 at 38 weeks gestation has been on L and D for the past 24 hours after her water broke. Microscopic evaluation of vaginal fluid reveals what appears to be a ferning pattern. The patient has no contractions. What is this? Good. Premature rupture of membranes or PROM. Okay, so abruptia placenta, I've talked about the risk factors. Again, you want to do an urgent c-section. So this lady has premature rupture of membranes, and she actually has premature prolonged rupture of membranes. You don't need to do a c-section for this, just sort of like give oxytocin. You want to move mom towards delivery of some sort. So let's talk about some alphabet soup. OB-GYN friends love your alphabet soup, okay? So there's something known as PROM. There's something known as PROM. PPROM. There's something known as double PPROM. Okay. So let's define those terms. I know. So PROM is a premature rupture of membranes. It just means membranes have ruptured. Okay. Before contractions have started. Okay. So if mom's water breaks, but no contractions, yeah, that's a premature rupture of membrane because normally you start contractions and boom, your water then breaks. But if your water breaks and no contractions, that's a premature rupture of membranes. Okay. So that's one. And in general, you make that diagnosis. You can do like the nitrosine paper test. So you can perform microscopy, see the ferrining pattern. Okay. Now, a PPROM or a preterm premature rupture of membranes, that's basically a rupture of membranes without rupture at less than 37 weeks. Right. So like preterm means less than 37 weeks. Right. So PPROM just means your water breaks. You haven't started contractions yet. And that happens before 37 weeks. That is preterm premature rupture of membranes. Now, PPPROM or prolonged preterm premature rupture of membranes, okay, is basically the P... prom definition uh applied to a female that her water is broken for more than so actually i'll be careful here uh so i'll let you ask your obi-gain like clerkship director this or whatever but i will say like some texts say if it's more than 24 hours it's prolonged some texts say if it's more than 20 hours it's prolonged so i'll let you uh figure that out on your own but i'm just gonna put 24 hours here uh that seemed to be pretty common when i look through different obi-gain texts okay now Some other quick things here. Choreo. Choreo amnionide. We talked about that, right? Remember, you give amp and gent for that. Remember, your CHE repeats. And then the classic presentation, they have talked about that in a previous slide. And again, I'll just repeat this abstruse scenario again, okay? If mom has ruptured, her water is broken for more than 18 hours, okay? Go ahead and give penicillin for GPS prophylaxis, okay? And then, again, I've talked about if you're less than 34 weeks, you want to go ahead and give steroids like dexamethasone to promote the fetal lung maturity. If you're less than 32 weeks, you want to give MAC to decrease the risk of cerebral palsy. And you don't want to do too many cervical checks, right? So you don't like push bugs up the vaginal canal. Okay. Now, preterm labor. You can actually make the... diagnosis with something called a fibronectin test, okay? But please don't confuse the fibronectin test for preterm labor with the nitrogen test for rupture of membranes, okay? The fibronectin test is like an early warning saying that maybe in like two weeks, a baby will come out, okay? It actually has a pretty good negative predictive value for preterm labor. Now, preterm labor, in general, right? If, again, if it's less than 34 weeks... You probably want to try to delay things a little, right? But you can't delay things for like, you can say, oh, let me push off this pregnancy for two weeks. Let me just give as much tocolytic as possible. No, you can't really do that, right? You can delay for like 24 to 48 hours, right? But in that time, you buy yourself some goodwill by giving steroids to promote fetal long maturity, okay? You give MAC for neuroprotection if it's less than 32 weeks, okay? And you can tocolyze the lady's uterus with like tributaline, it's a beta-2 agonist. I remember, right, if... Your cyclic AMP levels go up in smooth muscle. They cause relaxation, right? Because you have an inhibition of myosin-like chain kinase, okay? So, tributylene, rhododendrons, abetotroganase, they're very good at tocolytics. You can give MAG as a tocolytic, although it's, in general, it's not done. It's not indicated, pretty much. You could also give nifedipine. That could also help. And again, in the methacin, you want to exercise caution, right? Because after 32 weeks, you can close the ductus arteriosus prematurely. And that will be bad. Okay. And actually having a UTI or like bacterial vaginosis, this is a very high concept to know. They actually increase the risk of preterm labor. Okay. And then having like multiple pregnancies along with a prior history of a preterm labor is also a pretty big, those are pretty big risk factors for preterm labor. Okay. And having a bicornuate uterus where the paramezonephric ducts don't fuse. Okay. That's also an important risk factor for preterm labor. Okay, so given this info cluster, what do you think is going on, right? So hypoplastic fetal lungs, AFI is 2, right? So it's less than 5, so that's oligohydramnios. And the kid has facial, skin, and limb defects. What is that? Good. Potter sequence. Potter sequence. Okay. So bilateral renal agenesis. Okay. So oligohydramnios. You may perform an amnioinfusion, but it doesn't really help. These kids usually don't make it. Okay. Another cause of oligohydramnios, especially if they describe a newborn that has like a fullness on palpation of the suprapubic area, think about a posterior urethral valve. Okay, I talk about that in the Peds podcast, in the Peds vodcast, if you may. And then polyhydramnios, right, so think of anencephaly, neurotube defect, mom did not take a nofolate. Think of maternal diabetes. Okay, remember in anencephaly, the swallowing center is gone. Maternal diabetes, if a baby is hyperglycemic, it filters more sugar. at the level of the glomerulus, water goes alongside, so you have fetal polyuria, okay, so that can cause polyhydramnios. Duodenal atresia, right, so like double bubble sign on abdominal imaging, kid with a history of Down syndrome, okay, because nothing is going through the GI tract, again, you can get polyhydramnios with that, and then, so vaginal atresia, right, so like the bacterial association, that can also be a cause of polyhydramnios, because remember, kids, they pee, they swallow it, they pee, they swallow it, okay. Now, given the following info cluster, what's the most likely diagnosis, right? So, mom's size is greater than dates. Quad screen reveals elevated AFP and beta HCG, okay? An astute scientist observed splitting of the embryo on day 10 after fertilization occurred, right? This is not a very realistic question, but you get the point. Now, two babies are born. One is anemic and small for gestational age. The other is polycythemic, volume overloaded, and has signs of heart failure. What is this? Good, a twin-twin transfusion. It's a twin-twin transfusion. Awesome. Okay, so twin-twin transfusion syndrome, right? So this patient probably has a mono, delivered like mono-mono twins, okay? So here's the thing. This twinship, by day of split, probably want to go ahead and know that as well, okay? And you probably want to be able to recognize these twin ships on imaging. So I will encourage you to look up ultrasound pictures, okay? The way I remember it, I was just going fours, and then I remember the state of California, right? So C and A, okay? So what do I mean by that? You start with, so remember chorionic amniotic, okay? And then you start with di-di, okay? And then go to mono-di, and then you go to mono-mono. So like an MD is in the middle, mono-dye. So dye-dye, mono-dye, mono-mono. So dye-dye is if you get a dye-chorionic, dye-amniotic twin, if you have the split within the first four days, if it's from these 5 to 8, you get a mono-dye twin. If it's from these 9 to 12, you get a mono-mono twin. And then if it's after the 13, you're probably going to get the conjoined twins with that. And actually, the first time with these separate kids... Joined at the head. They actually did it here at Hopkins. Okay. Now, next question. 22-year-old female presents with lower abdominal pain and vaginal bleeding. Herr last menstrual period was seven weeks ago. She has a past medical history of PID. She smokes a pack of cigarettes on a daily basis. What is this? Good. Ectopic pregnancy. So I give you all the high-yield risk factors, right? So like PID, right? So she may have scarred the fallopian tubes or also called the oviduct. Smoke cigarettes, right? So smoking actually screws up your motility of those wispy things in the oviduct, okay? So this is an ectopic pregnancy, okay? And remember, the classic location of implantation, or the most common location, is in the fallopian tube. More specifically, the ampulla of the fallopian tube, okay? And there are certain big risk factors you want to remember, right? So like tubal ligation, history of tubal ligation, those things are not 100% perfect. prior history of an ectopic pregnancy that's actually a pretty big risk factor pid smoking i've talked about that and basically these scenarios are very easy on exams well not so much i'll talk about the not so much part in a bit but if a patient comes in you suspect an ectopic pregnancy and they tell you beta hcg is positive hemodynamically unstable your job is easy proceed to surgery okay call your ob-gyn well fellows to help you with that if the lady is and beta hcg is less than like five thousand You can actually give methotrexate. That could help. But if you have bad kidneys or a bad liver, probably avoid methotrexate. OK. And let me just say something real quick. Right. So if a lady is like hemodynamically stable, you suspect an atopic pregnancy, beta HCG is positive. On your exam, they could say the beta HCG is less than 1500. OK. If you see that low number, you potentially may not see a gestational sac on ultrasound, okay? So what you do is, you recheck in like two days, okay? If the beta-HCG is risen by like 67%, usually like if it's doubled, okay? That's a good sign. If it's still less than 1500, you just follow it up till it's more than 1500, and then you do an ultrasound, okay, to find where the baby is, to look for the gestational sac in the uterus. But if the number is greater than 1,500 and you don't see a gestational sac, okay, that's pretty diagnostic of an ectopic pregnancy, okay? Alternatively, if it's less than 1,500 but it does not rise appropriately, okay, that's also worrisome, right? That's worrisome for some kind of... the badness with the pregnancy. I mean, there's some stuff you'll do with progesterone and all that stuff, but that's, I think, probably a little too detailed, so I'll let you look that up on your own, okay? And there's a diagnostic algorithm. I'll let you look through this on your own, but I think if you understand the basics of what I described, you're probably good to go for your shelf. Okay, so let's continue this. So match the clinical presentation to the most likely diagnosis of spontaneous abortion. Okay, so before 20 weeks. Now, vaginal bleeding, closed os, viable fetus and ultrasound. What is this? Good. Threatened abortion. Now, vaginal bleeding, closed os, non-viable fetus and ultrasound. That's a missed abortion. Very good. Okay. Now, vaginal bleeding, an open os, intact gestational sac and ultrasound. Good. Inevitable abortion. Now, vaginal bleeding, open eyes, some products of conception and ultrasound. Some, not all. Good. That's an incomplete abortion. Now, vaginal bleeding, open eyes, no products of conception. What is that? Good, that's a complete abortion. Those are your answers. So in general, for spontaneous abortion, right, so again, less than 20 weeks, you do a DNC, a dilation and curatage, okay? You can also try to induce labor with your teratonic agent like oxytocin, okay? Now, you may see ethical scenarios, right, so like behavioral science questions if you're male in your exam, okay? In general, if a patient is stable, you may want to like express some kind of sympathy, provide support, but again, don't keep that baby in there for too long. So mom doesn't get septic and die as well, okay? Now, for a septic abortion, on your exam, you want to give broad spectrum and... antibiotics, okay, you want to perform an urgent DNC, right, so this is not the time to wait around for like 24 hours, no, a mom can become septic and die, okay, so you go ahead and do an urgent DNC and give broad-spec antibiotics, okay, after 20 weeks, it's no longer called a spontaneous abortion, it's called an IUFD, an intrauterine fetal demise, okay, in general, For these, you don't want to do a DNC. You want to do something called a DNE, okay? A dilation and evacuation, okay? Usually, you just try to induce labor so that the mom can give birth to the fetus. Don't do a C-section, okay? Resist the temptation to do a C-section, okay? The risk of the surgery does not go with the rewards, okay? So, they'll almost always give you that as an answer. Don't do that on your exam, okay? I'm sure there's probably exceptions to that rule, but for your exam, never do that. Okay, now a surclash, right? If a lady has a history of like painless, it's almost always in the second trimester on tests, right? So like painless second trimester pregnancy loss from cervical insufficiency, think about replacing a surclash, okay? And again, if you replace a surclash, that's a contraindication to exercise, okay? And the classic history there is a woman with a history of like a leap procedure for like, I don't know, like CIN1 or 2, whatever, like some kind of cervical pathology where they chopped off some tissue. Okay, so what's the most likely bug? Let's play a quick game, right? So let's do this really quick. We should be done fairly soon. So choroiditis, hydrocephalus, intracranial calcifications, cat later, you can give folate inhibitors. Good. Toxo, awesome. And if mom has the infection during pregnancy, you can give spironomycin. Now, slapped cheek rash, arthritis in adults, anemia slash vis-a-vis fetal hydrops. Slap check rash. It's a single-stranded DNA virus. Good. Parvo B19. Remember, it infects erythroid progenitors. Now, fetus born with scarred skin, hypoplastic limbs, life-threatening pneumonia. Mom had a generalized rash during pregnancy with vesicles and blisters in different stages of healing. This is herpes. Three. Good. Right. So, VZV. So, you could potentially give it cyclovir. Now, mom consumed daily meats, stillborn fetus, abscesses in the heart, liver, spleen, give ampicillin. Right. So, let's say mom consumes soft cheeses as well. Good. This is listeria. Okay. Granulomatosis infantis septica. Okay. Now, cataracts, deafness, machine-like murmur, ding, ding, ding, in a newborn, blueberry muffin rash. That's rubella. Good. Now, Periventricular, this is the buzzword, periventricular calcifications in the fetus and sensory neurohearing loss with jaundice and hepatosplenomegaly. This is CMV, very good, CMV, okay? So, if you see the buzzword, because CMV can present very similarly to toxo, okay? But the big thing they have to give you is calcifications around the ventricles, okay? Periventricular calcifications, stop reading the questions, that is CMV. Okay. Now, prevent infection by a scheduled C-section. Give AZT intrapartum during delivery. Good. That's HIV. Okay. Give a cycloveraprophylaxis starting at 36 weeks with history of infection. that's HSV, okay, now, classic exam scenario, they say, oh, vaginal delivery or C-section, okay, here's the big thing you want to look out for, if you see visible herpes lesions on the vagina, C-section, if you do not see lesions on the vagina, vaginal delivery, no lesions, vaginal delivery, lesions, C-section, okay, herpes, that's it, now, sloughed skin of the hands and feet, not hands and feet, sorry, let me correct that, of the hands and and feet. Okay. So slough skin of the hands and feet. Tons of nasal secretions in the neonate. Later in life, the kid has a permanent forehead, collapsed nasal bridge, Hutchinson's teeth, anterior tibial bone. Good. All right. So this is what? Syphilis. Right. So snuffles in a newborn think syphilis. And probably want to wear gloves while you're touching those snuffles. Okay. Because those things are tibial spirochetes. Okay. Now, uh, And remember, if a mom has syphilis in pregnancy, let's say she's penicillin allergic, what do you do? Do you give like a moxicillin or tetracycline? Well, you don't want to give a tetracycline, right? That's a teratogen. That's not a good idea. You want to give a moxicillin or a macrolid? No, you don't want to do those either, okay? Penicillin is the best drug for the treatment of syphilis in pregnancy. So if mom is panallergic, you desensitize her and still give the penicillin, okay? Now, what's the cause of any notable infection in the first 28 days of life? Good, right? Grubi strep, strep egalactiae. Awesome. So those are your answers. Now, next one. 67-year-old female presents with a one-year history of inogenital pruritus. Exam reveals large ivory white patches on the vulva with extension to the perineum. What's your next best step in management? I'm not asking for diagnosis. What is your next best step in management? Don't treat it. Good. What do you want to get? A biopsy. You want to get a biopsy, right? What is this? What would this most likely be? Lichen sclerosus. Very good. Okay, but it could also be cancer. Okay, so yes, it's probably lichen sclerosus. You give the high-potency topical steroid like clobetazole, but you also want to get a biopsy because you're trying to rule out vulvar cancer as well. Okay, now 60-year-old female presents with a large adnexal mass that is resected. A pathological specimen reveals samoma buddies, our famous laminated calcifications. Good. Awesome. Right. So this is a serosys adenoma, a serosys adenocarcinoma. Now, 39-year-old female presents with a one-year history of severe nexal pain for several days during her menstrual cycle. Diagnostic laparoscopy reveals what appears to be walled-off dark brown material. Good. So this is an endometrioma. Okay. Now a 15-year-old female presents to the ED. So remember your chocolate says, right? So 15-year-old female presents to the ED with severe sudden onset abdominal pain. And these kids are almost always studying for exams on the exam. Now, a large mass was populated in the abdomen by an OB-GYN doctor two years earlier. Today reveals teeth and hair. There's dermoid cysts. Good. And just real quick, let me just mention this now as a bonus for you. Okay. Teratomas, they tend to present as anterior mediastinal masses. Very high yield to know that. Anterior mediastinal masses. Contrast that with neurotumors, like a neuroblastoma that tends to present in the posterior mediastinum. Okay. Now, a 10-year-old female is being evaluated for precocious puberty. An ultrasound reveals, and it's a female, precocious puberty, right? So. Overactive female, think of it that way. An ultrasound reveals an ovarian mass. What is that? And I'll tell you that this potentially has Carl Exner bodies. Good, that's a granulosa cell tumor. Awesome. Now a 10-year-old female has been, and the tumor marker is estrogen, right? Obviously. A 10-year-old female has been evaluated for virilization. An ultrasound reveals an ovarian mass. Testosterone levels at 10x the upper limit of normal. What is that? Leading cell tumor. Good. Now, do early menarche, nulliparity, and late menopause confer an increase or decrease risk of ovarian cancer? Are you having more cycles or fewer cycles with these risk factors? More cycles. So does that increase your risk or decrease your risk of ovarian cancer? Increase. Good. Now, what are your tumor markers for yolk sac tumors? AFP. Good. AFP. Remember, that's also the tumor marker for hepatocellular carcinoma. How about ocorio carcinoma? Beta HCG. Good. Granulosa cell tumors. Estrogen. Fantastic. Okay. Now, a 43-year-old female schedules an appointment with her gynecologist three months after her annual checkup. checkup. She has felt irritable and moody for the past nine weeks and often has to change out of her nightgown as a result of severe night sweats. Physical exam is notable for mild pre-tibial edema. Vitals are notable for mild tachypnea. The patient is in a good relationship with her husband of 20 years. She has regular 30-day menstrual cycles, although she has recorded no flow for the past two cycles. What is the next best step in the management of this patient? Let you think about this for a second. I love this question. It's probably hypothyroid, right? So what do you want to check here? Do you want to reassure her? Is this like menopausal symptoms? Just trying to reassure the patient. This woman is amenoric. What do you want to get? Good. Bingo. Beta HCG. Okay. So this lady may be pregnant. Okay. Watch out for this scenario. They'll give you a question where it's kind of nondescript. It's a lady you're like, ah, it's probably not pregnant. No, you can be sure. Get a beta HCG. Okay. Now, male-female sexual development. I'll just go over this real quick. I'll probably stop in like five slides, and I'll let you go over the rest on your own because we're pretty close to the end. Although there'll probably be like 20 slides that I'll let you go over on your own. It's just more questions, okay? So if you're a guy, right, you have a Y chromosome, okay? So on your Y chromosome, you have the sex determining region of the Y, okay? So you get testes with that. That's what you get in that deal, okay? So you get testes. Testes contain two kinds of cells, right? So, lytic cells, they produce testosterone, okay? Testosterone can be converted by five alpha reductase to DHT, okay? And then DHT can virilize the external genitalia, okay? Now, the testosterone is a survival factor, if you may, for the Wolfian duct, okay? So, it promotes the differentiation of, like, the epididymis, the vas deferens, and your seminal vesicles, okay? Now, your stratoli cells, right, they produce AMH, okay? Like, malarian inhibiting factor or anti-malarian factor. or anti-mullerian hormone like ten names for the same thing basically anti mullerian hormone is anti to the mullerian duct okay so real quick I'll just say this here the mullerian duct is not the precursor to your ovaries okay but everything from the fallopian tubes down to the upper third of the vagina comes from the mullerian duct but if you're female no Y chromosome no SRY no testes okay the default programming for everyone is female okay but If you have a Y chromosome, then you go down the male pathway. Okay. So no testes, no testosterone, wolfian duct, doesn't survive, no DHT, you do not virilize your external genitalia. Okay. No Sertoli cells, no AMH. Okay. So no AMH, the malaria structures will develop. Okay. Because you're not killing the malaria duct. And just real quick, your Sertoli cells also make inhibin that can go back and negatively inhibit FSH secretion. So, let's do some questions involving primary amenorrhea. The definition is like, if you're 13 years old, no menses and no secondary sexual characteristics, or 16. If you have secondary sexual characteristics and no menses. So, what's your diagnosis here? So, 20-year-old female has breasts and a uterus, and the levels of all hormones are normal. What is this? What do you think? Levels of all hormones are normal. Okay. Good, good, good. Awesome, awesome, awesome. Excellent. Right. So levels of all hormones are normal. Right. So for this, you want to think about what? Right. So this may be what? What do you think? So she may have something obstructing the vagina, right? Like what? Impefrate hymen. Good. Now, 20-year-old female supermodel or hardcore athlete has breasts and a uterus. What should be true of her hormone levels? All low, right? Because again, if you're working out too hard, your body is probably not producing... Your HPG axis is shut down, right? So you have a hypogonadotropic hypogonadism. Now, breasts, no uterus. Testosterone levels are high. Good. AIS. Androgen Insensitivity Syndrome. Okay. Now, breasts, no uterus, and karyotype reveals a 46XX phenotype. Awesome. Good. Right. So, this is what? Malarian agenesis, right? The big word is like MRKH syndrome, right? So like Meyer or Ketansky-Kosterhausen syndrome, right? So you just have agenesis of your malarian duct, right? So everything from the fallopian tubes to the upper third of the vagina does not develop. Now, no breasts, but has a uterus, can't smell. Easy, right? Common syndrome. So that's a cause of hypogonadotropic hypogonadism as well. Now, short, low posterior hairline, widely spaced nipples. High blood pressures in the arms, low blood pressures in the legs, no breasts, no... has no breasts, has a uterus. Good, this is Turner's syndrome, okay? No breasts because they have low estrogen, right? If you see like polydeveloped breasts, think of no estrogen. If you see no uterus, if you see no secondary sexual characteristics, think of no testosterone or no activity of testosterone, okay? Now, a uterus, no breasts and visual field defects. Good, right? So hyperprolactinemia, okay? Like a craniofaringeoma or something, right? That could also be compressing the optic chiasm. So those are your answers. And this is a quick algorithm I put together. I think I'll go ahead and stop here. So we're on slide 134. I'll let you go over the remaining stuff. And I think if you have any questions, you can always send me a message or post a comment. I'll try to respond. But just a few more questions, some stuff with adenomyosis, with... contraception, pretty high-yield slides, I would say, endometrial cancer, vaginal infection, cervical cancer, screening, like some quick stuff on hormones, some high-yield factoids, my references, and all the best on your exam. Okay? Any questions? None? Okay, perfect. Thank you.